bstrandable NCLEX pharmacology 5 of 5

Five Rights

1-Right Patient
2-Right Drug
3-Right Dose
4-Right Route
5-Right Time

Microdrip

60 gtt/ml

Macrodrip

10-20 gtt/ml

Insulin Rotation Sites

Rotate clockwise

Insulin Injection Areas

Abdomen (2 inches from navel)
Upper arms
Thighs
Buttocks

IM Injection Sites

Glueteal (Buttocks), Deltoid, Vastus Lateralis (thigh)

Absorption Rates

Deltoid, and vastus lateralis- 2mL
Gluteal- up to 5mL

IM Syringe Sizes

1-1.5 inches long
19-22 gauge

Subq Injection Sites

Abdomen
Upper Hips
Upper Back
Lateral Thighs
Lateral Upper Arm

Subq Syringe Sizes

3/8 inch
1/2 inch
5/8 inch

Intradermal Syring Sizes

5-15 degree angle
1/4-5/8 Inch
25-29 gauge

1 Gallon

0

1 Quart

0

1 Pint

0

1 Cup

0

1 Tbsp

0

1 Tsp

0

1 Inch

0

Total volume X drop factor / Time in mins

#NAME?

Total volume to infuse / ml/ hour being infused

#NAME?

Total volume in mL / Number of hours

#NAME?

1) A nurse is caring for a client with hyperparathyroidism and notes that the client's serum calcium level is 13 mg/dL. Which medication should the nurse prepare to administer as prescribed to the client?
1. Calcium chloride
2. Calcium gluconate
3. Calcit

3. Calcitonin (Miacalcin)
Rationale:
The normal serum calcium level is 8.6 to 10.0 mg/dL. This client is experiencing hypercalcemia. Calcium gluconate and calcium chloride are medications used for the treatment of tetany, which occurs as a result of acute

2.) Oral iron supplements are prescribed for a 6-year-old child with iron deficiency anemia. The nurse instructs the mother to administer the iron with which best food item?
1. Milk
2. Water
3. Apple juice
4. Orange juice

4. Orange juice
Rationale:
Vitamin C increases the absorption of iron by the body. The mother should be instructed to administer the medication with a citrus fruit or a juice that is high in vitamin C. Milk may affect absorption of the iron. Water will no

3.) Salicylic acid is prescribed for a client with a diagnosis of psoriasis. The nurse monitors the client, knowing that which of the following would indicate the presence of systemic toxicity from this medication?
1. Tinnitus
2. Diarrhea
3. Constipation

1. Tinnitus
Rationale:
Salicylic acid is absorbed readily through the skin, and systemic toxicity (salicylism) can result. Symptoms include tinnitus, dizziness, hyperpnea, and psychological disturbances. Constipation and diarrhea are not associated with s

4.) The camp nurse asks the children preparing to swim in the lake if they have applied sunscreen. The nurse reminds the children that chemical sunscreens are most effective when applied:
1. Immediately before swimming
2. 15 minutes before exposure to the

4. At least 30 minutes before exposure to the sun
Rationale:
Sunscreens are most effective when applied at least 30 minutes before exposure to the sun so that they can penetrate the skin. All sunscreens should be reapplied after swimming or sweating.

5.) Mafenide acetate (Sulfamylon) is prescribed for the client with a burn injury. When applying the medication, the client complains of local discomfort and burning. Which of the following is the most appropriate nursing action?
1. Notifying the register

3. Informing the client that this is normal
Rationale:
Mafenide acetate is bacteriostatic for gram-negative and gram-positive organisms and is used to treat burns to reduce bacteria present in avascular tissues. The client should be informed that the medi

6.) The burn client is receiving treatments of topical mafenide acetate (Sulfamylon) to the site of injury. The nurse monitors the client, knowing that which of the following indicates that a systemic effect has occurred?
1.Hyperventilation
2.Elevated blo

1.Hyperventilation
Rationale:
Mafenide acetate is a carbonic anhydrase inhibitor and can suppress renal excretion of acid, thereby causing acidosis. Clients receiving this treatment should be monitored for signs of an acid-base imbalance (hyperventilation

7.) Isotretinoin is prescribed for a client with severe acne. Before the administration of this medication, the nurse anticipates that which laboratory test will be prescribed?
1. Platelet count
2. Triglyceride level
3. Complete blood count
4. White blood

2. Triglyceride level
Rationale:
Isotretinoin can elevate triglyceride levels. Blood triglyceride levels should be measured before treatment and periodically thereafter until the effect on the triglycerides has been evaluated. Options 1, 3, and 4 do not n

8.) A client with severe acne is seen in the clinic and the health care provider (HCP) prescribes isotretinoin. The nurse reviews the client's medication record and would contact the (HCP) if the client is taking which medication?
1. Vitamin A
2. Digoxin

1. Vitamin A
Rationale:
Isotretinoin is a metabolite of vitamin A and can produce generalized intensification of isotretinoin toxicity. Because of the potential for increased toxicity, vitamin A supplements should be discontinued before isotretinoin thera

9.) The nurse is applying a topical corticosteroid to a client with eczema. The nurse would monitor for the potential for increased systemic absorption of the medication if the medication were being applied to which of the following body areas?
1. Back
2.

2. Axilla
Rationale:
Topical corticosteroids can be absorbed into the systemic circulation. Absorption is higher from regions where the skin is especially permeable (scalp, axilla, face, eyelids, neck, perineum, genitalia), and lower from regions in which

10.) The clinic nurse is performing an admission assessment on a client. The nurse notes that the client is taking azelaic acid (Azelex). Because of the medication prescription, the nurse would suspect that the client is being treated for:
1. Acne
2. Ecze

1. Acne
Rationale:
Azelaic acid is a topical medication used to treat mild to moderate acne. The acid appears to work by suppressing the growth of Propionibacterium acnes and decreasing the proliferation of keratinocytes. Options 2, 3, and 4 are incorrect

11.) The health care provider has prescribed silver sulfadiazine (Silvadene) for the client with a partial-thickness burn, which has cultured positive for gram-negative bacteria. The nurse is reinforcing information to the client about the medication. Whi

3. "The medication will permanently stain my skin."
Rationale:
Silver sulfadiazine (Silvadene) is an antibacterial that has a broad spectrum of activity against gram-negative bacteria, gram-positive bacteria, and yeast. It is applied directly to the wound

12.) A nurse is caring for a client who is receiving an intravenous (IV) infusion of an antineoplastic medication. During the infusion, the client complains of pain at the insertion site. During an inspection of the site, the nurse notes redness and swell

1. Notify the registered nurse.
Rationale:
When antineoplastic medications (Chemotheraputic Agents) are administered via IV, great care must be taken to prevent the medication from escaping into the tissues surrounding the injection site, because pain, ti

13.) The client with squamous cell carcinoma of the larynx is receiving bleomycin intravenously. The nurse caring for the client anticipates that which diagnostic study will be prescribed?
1. Echocardiography
2. Electrocardiography
3. Cervical radiography

4. Pulmonary function studies
Rationale:
Bleomycin is an antineoplastic medication (Chemotheraputic Agents) that can cause interstitial pneumonitis, which can progress to pulmonary fibrosis. Pulmonary function studies along with hematological, hepatic, an

14.) The client with acute myelocytic leukemia is being treated with busulfan (Myleran). Which laboratory value would the nurse specifically monitor during treatment with this medication?
1. Clotting time
2. Uric acid level
3. Potassium level
4. Blood glu

2. Uric acid level
Rationale:
Busulfan (Myleran) can cause an increase in the uric acid level. Hyperuricemia can produce uric acid nephropathy, renal stones, and acute renal failure. Options 1, 3, and 4 are not specifically related to this medication.

15.) The client with small cell lung cancer is being treated with etoposide (VePesid). The nurse who is assisting in caring for the client during its administration understands that which side effect is specifically associated with this medication?
1. Alo

4. Orthostatic hypotension
Rationale:
A side effect specific to etoposide is orthostatic hypotension. The client's blood pressure is monitored during the infusion. Hair loss occurs with nearly all the antineoplastic medications. Chest pain and pulmonary f

16.) The clinic nurse is reviewing a teaching plan for the client receiving an antineoplastic medication. When implementing the plan, the nurse tells the client:
1. To take aspirin (acetylsalicylic acid) as needed for headache
2. Drink beverages containin

3. Consult with health care providers (HCPs) before receiving immunizations
Rationale:
Because antineoplastic medications lower the resistance of the body, clients must be informed not to receive immunizations without a HCP's approval. Clients also need t

17.) The client with ovarian cancer is being treated with vincristine (Oncovin). The nurse monitors the client, knowing that which of the following indicates a side effect specific to this medication?
1. Diarrhea
2. Hair loss
3. Chest pain
4. Numbness and

4. Numbness and tingling in the fingers and toes
Rationale:
A side effect specific to vincristine is peripheral neuropathy, which occurs in almost every client. Peripheral neuropathy can be manifested as numbness and tingling in the fingers and toes. Depr

18.) The nurse is reviewing the history and physical examination of a client who will be receiving asparaginase (Elspar), an antineoplastic agent. The nurse consults with the registered nurse regarding the administration of the medication if which of the

1. Pancreatitis
Rationale:
Asparaginase (Elspar) is contraindicated if hypersensitivity exists, in pancreatitis, or if the client has a history of pancreatitis. The medication impairs pancreatic function and pancreatic function tests should be performed b

19.) Tamoxifen is prescribed for the client with metastatic breast carcinoma. The nurse understands that the primary action of this medication is to:
1. Increase DNA and RNA synthesis.
2. Promote the biosynthesis of nucleic acids.
3. Increase estrogen con

4. Compete with estradiol for binding to estrogen in tissues containing high concentrations of receptors.
Rationale:
Tamoxifen is an antineoplastic medication that competes with estradiol for binding to estrogen in tissues containing high concentrations o

20.) The client with metastatic breast cancer is receiving tamoxifen. The nurse specifically monitors which laboratory value while the client is taking this medication?
1. Glucose level
2. Calcium level
3. Potassium level
4. Prothrombin time

2. Calcium level
Rationale:
Tamoxifen may increase calcium, cholesterol, and triglyceride levels. Before the initiation of therapy, a complete blood count, platelet count, and serum calcium levels should be assessed. These blood levels, along with cholest

21.) A nurse is assisting with caring for a client with cancer who is receiving cisplatin. Select the adverse effects that the nurse monitors for that are associated with this medication. Select all that apply.
1. Tinnitus
2. Ototoxicity
3. Hyperkalemia
4

1. Tinnitus
2. Ototoxicity
5. Nephrotoxicity
6. Hypomagnesemia
Rationale:
Cisplatin is an alkylating medication. Alkylating medications are cell cycle phase-nonspecific medications that affect the synthesis of DNA by causing the cross-linking of DNA to in

22.) A nurse is caring for a client after thyroidectomy and notes that calcium gluconate is prescribed for the client. The nurse determines that this medication has been prescribed to:
1. Treat thyroid storm.
2. Prevent cardiac irritability.
3. Treat hypo

3. Treat hypocalcemic tetany.
Rationale:
Hypocalcemia can develop after thyroidectomy if the parathyroid glands are accidentally removed or injured during surgery. Manifestations develop 1 to 7 days after surgery. If the client develops numbness and tingl

23.) A client who has been newly diagnosed with diabetes mellitus has been stabilized with daily insulin injections. Which information should the nurse teach when carrying out plans for discharge?
1. Keep insulin vials refrigerated at all times.
2. Rotate

2. Rotate the insulin injection sites systematically.
Rationale:
Insulin dosages should not be adjusted or increased before unusual exercise. If acetone is found in the urine, it may possibly indicate the need for additional insulin. To minimize the disco

24.) A nurse is reinforcing teaching for a client regarding how to mix regular insulin and NPH insulin in the same syringe. Which of the following actions, if performed by the client, indicates the need for further teaching?
1. Withdraws the NPH insulin f

1. Withdraws the NPH insulin first
Rationale:
When preparing a mixture of regular insulin with another insulin preparation, the regular insulin is drawn into the syringe first. This sequence will avoid contaminating the vial of regular insulin with insuli

25.) A home care nurse visits a client recently diagnosed with diabetes mellitus who is taking Humulin NPH insulin daily. The client asks the nurse how to store the unopened vials of insulin. The nurse tells the client to:
1. Freeze the insulin.
2. Refrig

2. Refrigerate the insulin.
Rationale:
Insulin in unopened vials should be stored under refrigeration until needed. Vials should not be frozen. When stored unopened under refrigeration, insulin can be used up to the expiration date on the vial. Options 1,

26.) Glimepiride (Amaryl) is prescribed for a client with diabetes mellitus. A nurse reinforces instructions for the client and tells the client to avoid which of the following while taking this medication?
1. Alcohol
2. Organ meats
3. Whole-grain cereals

1. Alcohol
Rationale:
When alcohol is combined with glimepiride (Amaryl), a disulfiram-like reaction may occur. This syndrome includes flushing, palpitations, and nausea. Alcohol can also potentiate the hypoglycemic effects of the medication. Clients need

27.) Sildenafil (Viagra) is prescribed to treat a client with erectile dysfunction. A nurse reviews the client's medical record and would question the prescription if which of the following is noted in the client's history?
1. Neuralgia
2. Insomnia
3. Use

3. Use of nitroglycerin
Rationale:
Sildenafil (Viagra) enhances the vasodilating effect of nitric oxide in the corpus cavernosum of the penis, thus sustaining an erection. Because of the effect of the medication, it is contraindicated with concurrent use

28.) The health care provider (HCP) prescribes exenatide (Byetta) for a client with type 1 diabetes mellitus who takes insulin. The nurse knows that which of the following is the appropriate intervention?
1. The medication is administered within 60 minute

2. The medication is withheld and the HCP is called to question the prescription for the client.
Rationale:
Exenatide (Byetta) is an incretin mimetic used for type 2 diabetes mellitus only. It is not recommended for clients taking insulin. Hence, the nurs

29.) A client is taking Humulin NPH insulin daily every morning. The nurse reinforces instructions for the client and tells the client that the most likely time for a hypoglycemic reaction to occur is:
1. 2 to 4 hours after administration
2. 4 to 12 hours

2. 4 to 12 hours after administration
Rationale:
Humulin NPH is an intermediate-acting insulin. The onset of action is 1.5 hours, it peaks in 4 to 12 hours, and its duration of action is 24 hours. Hypoglycemic reactions most likely occur during peak time.

30.) A client with diabetes mellitus visits a health care clinic. The client's diabetes mellitus previously had been well controlled with glyburide (DiaBeta) daily, but recently the fasting blood glucose level has been 180 to 200 mg/dL. Which medication,

1. Prednisone
Rationale:
Prednisone may decrease the effect of oral hypoglycemics, insulin, diuretics, and potassium supplements. Option 2, a monoamine oxidase inhibitor, and option 3, a ?-blocker, have their own intrinsic hypoglycemic activity. Option 4

31.) A community health nurse visits a client at home. Prednisone 10 mg orally daily has been prescribed for the client and the nurse reinforces teaching for the client about the medication. Which statement, if made by the client, indicates that further t

1. "I can take aspirin or my antihistamine if I need it."
Rationale:
Aspirin and other over-the-counter medications should not be taken unless the client consults with the health care provider (HCP). The client needs to take the medication at the same tim

32.) Desmopressin acetate (DDAVP) is prescribed for the treatment of diabetes insipidus. The nurse monitors the client after medication administration for which therapeutic response?
1. Decreased urinary output
2. Decreased blood pressure
3. Decreased per

1. Decreased urinary output
Rationale:
Desmopressin promotes renal conservation of water. The hormone carries out this action by acting on the collecting ducts of the kidney to increase their permeability to water, which results in increased water reabsor

33.) The home health care nurse is visiting a client who was recently diagnosed with type 2 diabetes mellitus. The client is prescribed repaglinide (Prandin) and metformin (Glucophage) and asks the nurse to explain these medications. The nurse should rein

1. Diarrhea can occur secondary to the metformin.
2. The repaglinide is not taken if a meal is skipped.
3. The repaglinide is taken 30 minutes before eating.
4. Candy or another simple sugar is carried and used to treat mild hypoglycemia episodes.
Rationa

34.) A client with Crohn's disease is scheduled to receive an infusion of infliximab (Remicade). The nurse assisting in caring for the client should take which action to monitor the effectiveness of treatment?
1. Monitoring the leukocyte count for 2 days

2. Checking the frequency and consistency of bowel movements
Rationale:
The principal manifestations of Crohn's disease are diarrhea and abdominal pain. Infliximab (Remicade) is an immunomodulator that reduces the degree of inflammation in the colon, ther

35.) The client has a PRN prescription for loperamide hydrochloride (Imodium). The nurse understands that this medication is used for which condition?
1. Constipation
2. Abdominal pain
3. An episode of diarrhea
4. Hematest-positive nasogastric tube draina

3. An episode of diarrhea
Rationale:
Loperamide is an antidiarrheal agent. It is used to manage acute and also chronic diarrhea in conditions such as inflammatory bowel disease. Loperamide also can be used to reduce the volume of drainage from an ileostom

36.) The client has a PRN prescription for ondansetron (Zofran). For which condition should this medication be administered to the postoperative client?
1. Paralytic ileus
2. Incisional pain
3. Urinary retention
4. Nausea and vomiting

4. Nausea and vomiting
Rationale:
Ondansetron is an antiemetic used to treat postoperative nausea and vomiting, as well as nausea and vomiting associated with chemotherapy. The other options are incorrect.

37.) The client has begun medication therapy with pancrelipase (Pancrease MT). The nurse evaluates that the medication is having the optimal intended benefit if which effect is observed?
1. Weight loss
2. Relief of heartburn
3. Reduction of steatorrhea
4.

3. Reduction of steatorrhea
Rationale:
Pancrelipase (Pancrease MT) is a pancreatic enzyme used in clients with pancreatitis as a digestive aid. The medication should reduce the amount of fatty stools (steatorrhea). Another intended effect could be improve

38.) An older client recently has been taking cimetidine (Tagamet). The nurse monitors the client for which most frequent central nervous system side effect of this medication?
1. Tremors
2. Dizziness
3. Confusion
4. Hallucinations

3. Confusion
Rationale:
Cimetidine is a histamine 2 (H2)-receptor antagonist. Older clients are especially susceptible to central nervous system side effects of cimetidine. The most frequent of these is confusion. Less common central nervous system side e

39.) The client with a gastric ulcer has a prescription for sucralfate (Carafate), 1 g by mouth four times daily. The nurse schedules the medication for which times?
1. With meals and at bedtime
2. Every 6 hours around the clock
3. One hour after meals an

4. One hour before meals and at bedtime
Rationale:
Sucralfate is a gastric protectant. The medication should be scheduled for administration 1 hour before meals and at bedtime. The medication is timed to allow it to form a protective coating over the ulce

40.) The client who chronically uses nonsteroidal anti-inflammatory drugs has been taking misoprostol (Cytotec). The nurse determines that the medication is having the intended therapeutic effect if which of the following is noted?
1. Resolved diarrhea
2.

2. Relief of epigastric pain
Rationale:
The client who chronically uses nonsteroidal anti-inflammatory drugs (NSAIDs) is prone to gastric mucosal injury. Misoprostol is a gastric protectant and is given specifically to prevent this occurrence. Diarrhea ca

41.) The client has been taking omeprazole (Prilosec) for 4 weeks. The ambulatory care nurse evaluates that the client is receiving optimal intended effect of the medication if the client reports the absence of which symptom?
1. Diarrhea
2. Heartburn
3. F

2. Heartburn
Rationale:
Omeprazole is a proton pump inhibitor classified as an antiulcer agent. The intended effect of the medication is relief of pain from gastric irritation, often called heartburn by clients. Omeprazole is not used to treat the conditi

42.) A client with a peptic ulcer is diagnosed with a Helicobacter pylori infection. The nurse is reinforcing teaching for the client about the medications prescribed, including clarithromycin (Biaxin), esomeprazole (Nexium), and amoxicillin (Amoxil). Whi

3. "The medications will kill the bacteria and stop the acid production."
Rationale:
Triple therapy for Helicobacter pylori infection usually includes two antibacterial drugs and a proton pump inhibitor. Clarithromycin and amoxicillin are antibacterials.

43.) A histamine (H2)-receptor antagonist will be prescribed for a client. The nurse understands that which medications are H2-receptor antagonists? Select all that apply.
1. Nizatidine (Axid)
2. Ranitidine (Zantac)
3. Famotidine (Pepcid)
4. Cimetidine (T

1. Nizatidine (Axid)
2. Ranitidine (Zantac)
3. Famotidine (Pepcid)
4. Cimetidine (Tagamet)
Rationale:
H2-receptor antagonists suppress secretion of gastric acid, alleviate symptoms of heartburn, and assist in preventing complications of peptic ulcer disea

44.) A client is receiving acetylcysteine (Mucomyst), 20% solution diluted in 0.9% normal saline by nebulizer. The nurse should have which item available for possible use after giving this medication?
1. Ambu bag
2. Intubation tray
3. Nasogastric tube
4.

4. Suction equipment
Rationale:
Acetylcysteine can be given orally or by nasogastric tube to treat acetaminophen overdose, or it may be given by inhalation for use as a mucolytic. The nurse administering this medication as a mucolytic should have suction

45.) A client has a prescription to take guaifenesin (Humibid) every 4 hours, as needed. The nurse determines that the client understands the most effective use of this medication if the client states that he or she will:
1. Watch for irritability as a si

2. Take the tablet with a full glass of water.
Rationale:
Guaifenesin is an expectorant. It should be taken with a full glass of water to decrease viscosity of secretions. Sustained-release preparations should not be broken open, crushed, or chewed. The m

46.) A postoperative client has received a dose of naloxone hydrochloride for respiratory depression shortly after transfer to the nursing unit from the postanesthesia care unit. After administration of the medication, the nurse checks the client for:
1.

3. Sudden increase in pain
Rationale:
Naloxone hydrochloride is an antidote to opioids and may also be given to the postoperative client to treat respiratory depression. When given to the postoperative client for respiratory depression, it may also revers

47.) A client has been taking isoniazid (INH) for 2 months. The client complains to a nurse about numbness, paresthesias, and tingling in the extremities. The nurse interprets that the client is experiencing:
1. Hypercalcemia
2. Peripheral neuritis
3. Sma

2. Peripheral neuritis
Rationale:
A common side effect of the TB drug INH is peripheral neuritis. This is manifested by numbness, tingling, and paresthesias in the extremities. This side effect can be minimized by pyridoxine (vitamin B6) intake. Options 1

48.) A client is to begin a 6-month course of therapy with isoniazid (INH). A nurse plans to teach the client to:
1. Drink alcohol in small amounts only.
2. Report yellow eyes or skin immediately.
3. Increase intake of Swiss or aged cheeses.
4. Avoid vita

2. Report yellow eyes or skin immediately.
Rationale:
INH is hepatotoxic, and therefore the client is taught to report signs and symptoms of hepatitis immediately (which include yellow skin and sclera). For the same reason, alcohol should be avoided durin

49.) A client has been started on long-term therapy with rifampin (Rifadin). A nurse teaches the client that the medication:
1. Should always be taken with food or antacids
2. Should be double-dosed if one dose is forgotten
3. Causes orange discoloration

3. Causes orange discoloration of sweat, tears, urine, and feces
Rationale:
Rifampin should be taken exactly as directed as part of TB therapy. Doses should not be doubled or skipped. The client should not stop therapy until directed to do so by a health

50.) A nurse has given a client taking ethambutol (Myambutol) information about the medication. The nurse determines that the client understands the instructions if the client states that he or she will immediately report:
1. Impaired sense of hearing
2.

2. Problems with visual acuity
Rationale:
Ethambutol causes optic neuritis, which decreases visual acuity and the ability to discriminate between the colors red and green. This poses a potential safety hazard when a client is driving a motor vehicle. The

51.) Cycloserine (Seromycin) is added to the medication regimen for a client with tuberculosis. Which of the following would the nurse include in the client-teaching plan regarding this medication?
1. To take the medication before meals
2. To return to th

2. To return to the clinic weekly for serum drug-level testing
Rationale:
Cycloserine (Seromycin) is an antitubercular medication that requires weekly serum drug level determinations to monitor for the potential of neurotoxicity. Serum drug levels lower t

52.) A client with tuberculosis is being started on antituberculosis therapy with isoniazid (INH). Before giving the client the first dose, a nurse ensures that which of the following baseline studies has been completed?
1. Electrolyte levels
2. Coagulati

3. Liver enzyme levels
Rationale:
INH therapy can cause an elevation of hepatic enzyme levels and hepatitis. Therefore, liver enzyme levels are monitored when therapy is initiated and during the first 3 months of therapy. They may be monitored longer in t

53.) Rifabutin (Mycobutin) is prescribed for a client with active Mycobacterium avium complex (MAC) disease and tuberculosis. The nurse monitors for which side effects of the medication? Select all that apply.
1. Signs of hepatitis
2. Flu-like syndrome
3.

1. Signs of hepatitis
2. Flu-like syndrome
3. Low neutrophil count
5. Ocular pain or blurred vision
Rationale:
Rifabutin (Mycobutin) may be prescribed for a client with active MAC disease and tuberculosis. It inhibits mycobacterial DNA-dependent RNA polym

54.) A nurse reinforces discharge instructions to a postoperative client who is taking warfarin sodium (Coumadin). Which statement, if made by the client, reflects the need for further teaching?
1. "I will take my pills every day at the same time."
2. "I

4. "I will take Ecotrin (enteric-coated aspirin) for my headaches because it is coated."
Rationale:
Ecotrin is an aspirin-containing product and should be avoided. Alcohol consumption should be avoided by a client taking warfarin sodium. Taking prescribed

55.) A client who is receiving digoxin (Lanoxin) daily has a serum potassium level of 3.0 mEq/L and is complaining of anorexia. A health care provider prescribes a digoxin level to rule out digoxin toxicity. A nurse checks the results, knowing that which

2.) 0.5 to 2 ng/mL
Rationale:
Therapeutic levels for digoxin range from 0.5 to 2 ng/mL. Therefore, options 1, 3, and 4 are incorrect.

56.) Heparin sodium is prescribed for the client. The nurse expects that the health care provider will prescribe which of the following to monitor for a therapeutic effect of the medication?
1. Hematocrit level
2. Hemoglobin level
3. Prothrombin time (PT)

4. Activated partial thromboplastin time (aPTT)
Rationale:
The PT will assess for the therapeutic effect of warfarin sodium (Coumadin) and the aPTT will assess the therapeutic effect of heparin sodium. Heparin sodium doses are determined based on these la

57.) A nurse is monitoring a client who is taking propranolol (Inderal LA). Which data collection finding would indicate a potential serious complication associated with propranolol?
1. The development of complaints of insomnia
2. The development of audib

2. The development of audible expiratory wheezes
Rationale:
Audible expiratory wheezes may indicate a serious adverse reaction, bronchospasm. ?-Blockers may induce this reaction, particularly in clients with chronic obstructive pulmonary disease or asthma

58.) Isosorbide mononitrate (Imdur) is prescribed for a client with angina pectoris. The client tells the nurse that the medication is causing a chronic headache. The nurse appropriately suggests that the client:
1. Cut the dose in half.
2. Discontinue th

3. Take the medication with food.
Rationale:
Isosorbide mononitrate is an antianginal medication. Headache is a frequent side effect of isosorbide mononitrate and usually disappears during continued therapy. If a headache occurs during therapy, the client

59.) A client is diagnosed with an acute myocardial infarction and is receiving tissue plasminogen activator, alteplase (Activase, tPA). Which action is a priority nursing intervention?
1. Monitor for renal failure.
2. Monitor psychosocial status.
3. Moni

3. Monitor for signs of bleeding.
Rationale:
Tissue plasminogen activator is a thrombolytic. Hemorrhage is a complication of any type of thrombolytic medication. The client is monitored for bleeding. Monitoring for renal failure and monitoring the client'

60.) A nurse is planning to administer hydrochlorothiazide (HydroDIURIL) to a client. The nurse understands that which of the following are concerns related to the administration of this medication?
1. Hypouricemia, hyperkalemia
2. Increased risk of osteo

3. Hypokalemia, hyperglycemia, sulfa allergy
Rationale:
Thiazide diuretics such as hydrochlorothiazide are sulfa-based medications, and a client with a sulfa allergy is at risk for an allergic reaction. Also, clients are at risk for hypokalemia, hyperglyc

61.) A home health care nurse is visiting a client with elevated triglyceride levels and a serum cholesterol level of 398 mg/dL. The client is taking cholestyramine (Questran). Which of the following statements, if made by the client, indicates the need f

4. "I'll continue my nicotinic acid from the health food store."
Rationale:
Nicotinic acid, even an over-the-counter form, should be avoided because it may lead to liver abnormalities. All lipid-lowering medications also can cause liver abnormalities, so

62.) A client is on nicotinic acid (niacin) for hyperlipidemia and the nurse provides instructions to the client about the medication. Which statement by the client would indicate an understanding of the instructions?
1. "It is not necessary to avoid the

4. "Ibuprofen (Motrin) taken 30 minutes before the nicotinic acid should decrease the flushing."
Rationale:
Flushing is a side effect of this medication. Aspirin or a nonsteroidal anti-inflammatory drug can be taken 30 minutes before taking the medication

63.) A client with coronary artery disease complains of substernal chest pain. After checking the client's heart rate and blood pressure, a nurse administers nitroglycerin, 0.4 mg, sublingually. After 5 minutes, the client states, "My chest still hurts.

2. Contact the registered nurse.
4. Assess the client's pain level.
5. Check the client's blood pressure.
6. Administer a second nitroglycerin, 0.4 mg, sublingually.
Rationale:
The usual guideline for administering nitroglycerin tablets for a hospitalized

64.) Nalidixic acid (NegGram) is prescribed for a client with a urinary tract infection. On review of the client's record, the nurse notes that the client is taking warfarin sodium (Coumadin) daily. Which prescription should the nurse anticipate for this

2. A decrease in the warfarin sodium (Coumadin) dosage
Rationale:
Nalidixic acid can intensify the effects of oral anticoagulants by displacing these agents from binding sites on plasma protein. When an oral anticoagulant is combined with nalidixic acid,

65.) A nurse is reinforcing discharge instructions to a client receiving sulfisoxazole. Which of the following should be included in the list of instructions?
1. Restrict fluid intake.
2. Maintain a high fluid intake.
3. If the urine turns dark brown, cal

2. Maintain a high fluid intake.
Rationale:
Each dose of sulfisoxazole should be administered with a full glass of water, and the client should maintain a high fluid intake. The medication is more soluble in alkaline urine. The client should not be instru

66.) Trimethoprim-sulfamethoxazole (TMP-SMZ) is prescribed for a client. A nurse should instruct the client to report which symptom if it developed during the course of this medication therapy?
1. Nausea
2. Diarrhea
3. Headache
4. Sore throat

4. Sore throat
Rationale:
Clients taking trimethoprim-sulfamethoxazole (TMP-SMZ) should be informed about early signs of blood disorders that can occur from this medication. These include sore throat, fever, and pallor, and the client should be instructed

67.) Phenazopyridine hydrochloride (Pyridium) is prescribed for a client for symptomatic relief of pain resulting from a lower urinary tract infection. The nurse reinforces to the client:
1. To take the medication at bedtime
2. To take the medication befo

4. That a reddish orange discoloration of the urine may occur
Rationale:
The nurse should instruct the client that a reddish-orange discoloration of urine may occur. The nurse also should instruct the client that this discoloration can stain fabric. The m

68.) Bethanechol chloride (Urecholine) is prescribed for a client with urinary retention. Which disorder would be a contraindication to the administration of this medication?
1. Gastric atony
2. Urinary strictures
3. Neurogenic atony
4. Gastroesophageal r

2. Urinary strictures
Rationale:
Bethanechol chloride (Urecholine) can be harmful to clients with urinary tract obstruction or weakness of the bladder wall. The medication has the ability to contract the bladder and thereby increase pressure within the ur

69.) A nurse who is administering bethanechol chloride (Urecholine) is monitoring for acute toxicity associated with the medication. The nurse checks the client for which sign of toxicity?
1. Dry skin
2. Dry mouth
3. Bradycardia
4. Signs of dehydration

3. Bradycardia
Rationale:
Toxicity (overdose) produces manifestations of excessive muscarinic stimulation such as salivation, sweating, involuntary urination and defecation, bradycardia, and severe hypotension. Treatment includes supportive measures and t

70.) Oxybutynin chloride (Ditropan XL) is prescribed for a client with neurogenic bladder. Which sign would indicate a possible toxic effect related to this medication?
1. Pallor
2. Drowsiness
3. Bradycardia
4. Restlessness

4. Restlessness
Rationale:
Toxicity (overdosage) of this medication produces central nervous system excitation, such as nervousness, restlessness, hallucinations, and irritability. Other signs of toxicity include hypotension or hypertension, confusion, ta

71.) After kidney transplantation, cyclosporine (Sand immune) is prescribed for a client. Which laboratory result would indicate an adverse effect from the use of this medication?
1. Decreased creatinine level
2. Decreased hemoglobin level
3. Elevated blo

3. Elevated blood urea nitrogen level
Rationale:
Nephrotoxicity can occur from the use of cyclosporine (Sandimmune). Nephrotoxicity is evaluated by monitoring for elevated blood urea nitrogen (BUN) and serum creatinine levels. Cyclosporine is an immunosup

72.) Cinoxacin (Cinobac), a urinary antiseptic, is prescribed for the client. The nurse reviews the client's medical record and should contact the health care provider (HCP) regarding which documented finding to verify the prescription? Refer to chart.
1.

1. Renal insufficiency
Rationale:
Cinoxacin should be administered with caution in clients with renal impairment. The dosage should be reduced, and failure to do so could result in accumulation of cinoxacin to toxic levels. Therefore the nurse would verif

73.) A client with myasthenia gravis is suspected of having cholinergic crisis. Which of the following indicate that this crisis exists?
1. Ataxia
2. Mouth sores
3. Hypotension
4. Hypertension

4. Hypertension
Rationale:
Cholinergic crisis occurs as a result of an overdose of medication. Indications of cholinergic crisis include gastrointestinal disturbances, nausea, vomiting, diarrhea, abdominal cramps, increased salivation and tearing, miosis,

74.) A client with myasthenia gravis is receiving pyridostigmine (Mestinon). The nurse monitors for signs and symptoms of cholinergic crisis caused by overdose of the medication. The nurse checks the medication supply to ensure that which medication is av

2. Atropine sulfate
Rationale:
The antidote for cholinergic crisis is atropine sulfate. Vitamin K is the antidote for warfarin (Coumadin). Protamine sulfate is the antidote for heparin, and acetylcysteine (Mucomyst) is the antidote for acetaminophen (Tyle

75.) A client with myasthenia gravis becomes increasingly weak. The health care provider prepares to identify whether the client is reacting to an overdose of the medication (cholinergic crisis) or increasing severity of the disease (myasthenic crisis). A

4. A temporary worsening of the condition
Rationale:
An edrophonium (Enlon) injection, a cholinergic drug, makes the client in cholinergic crisis temporarily worse. This is known as a negative test. An improvement of weakness would occur if the client wer

76.) Carbidopa-levodopa (Sinemet) is prescribed for a client with Parkinson's disease, and the nurse monitors the client for adverse reactions to the medication. Which of the following indicates that the client is experiencing an adverse reaction?
1. Prur

4. Impaired voluntary movements
Rationale:
Dyskinesia and impaired voluntary movement may occur with high levodopa dosages. Nausea, anorexia, dizziness, orthostatic hypotension, bradycardia, and akinesia (the temporary muscle weakness that lasts 1 minute

77.) Phenytoin (Dilantin), 100 mg orally three times daily, has been prescribed for a client for seizure control. The nurse reinforces instructions regarding the medication to the client. Which statement by the client indicates an understanding of the ins

1. "I will use a soft toothbrush to brush my teeth."
Rationale:
Phenytoin (Dilantin) is an anticonvulsant. Gingival hyperplasia, bleeding, swelling, and tenderness of the gums can occur with the use of this medication. The client needs to be taught good o

78.) A client is taking phenytoin (Dilantin) for seizure control and a sample for a serum drug level is drawn. Which of the following indicates a therapeutic serum drug range?
1. 5 to 10 mcg/mL
2. 10 to 20 mcg/mL
3. 20 to 30 mcg/mL
4. 30 to 40 mcg/mL

2. 10 to 20 mcg/mL
Rationale:
The therapeutic serum drug level range for phenytoin (Dilantin) is 10 to 20 mcg/mL.
*
A helpful hint may be to remember that the theophylline therapeutic range and the acetaminophen (Tylenol) therapeutic range are the same as

79.) Ibuprofen (Advil) is prescribed for a client. The nurse tells the client to take the medication:
1. With 8 oz of milk
2. In the morning after arising
3. 60 minutes before breakfast
4. At bedtime on an empty stomach

1. With 8 oz of milk
Rationale:
Ibuprofen is a nonsteroidal anti-inflammatory drug (NSAID). NSAIDs should be given with milk or food to prevent gastrointestinal irritation. Options 2, 3, and 4 are incorrect.

80.) A nurse is caring for a client who is taking phenytoin (Dilantin) for control of seizures. During data collection, the nurse notes that the client is taking birth control pills. Which of the following information should the nurse provide to the clien

3. The potential for decreased effectiveness of the birth control pills exists while taking phenytoin (Dilantin).
Rationale:
Phenytoin (Dilantin) enhances the rate of estrogen metabolism, which can decrease the effectiveness of some birth control pills. O

81.) A client with trigeminal neuralgia is being treated with carbamazepine (Tegretol). Which laboratory result would indicate that the client is experiencing an adverse reaction to the medication?
1. Sodium level, 140 mEq/L
2. Uric acid level, 5.0 mg/dL

3. White blood cell count, 3000 cells/mm3
Rationale:
Adverse effects of carbamazepine (Tegretol) appear as blood dyscrasias, including aplastic anemia, agranulocytosis, thrombocytopenia, leukopenia, cardiovascular disturbances, thrombophlebitis, dysrhythm

82.) A client is receiving meperidine hydrochloride (Demerol) for pain. Which of the following are side effects of this medication. Select all that apply.
1. Diarrhea
2. Tremors
3. Drowsiness
4. Hypotension
5. Urinary frequency
6. Increased respiratory ra

2. Tremors
3. Drowsiness
4. Hypotension
Rationale:
Meperidine hydrochloride is an opioid analgesic. Side effects include respiratory depression, drowsiness, hypotension, constipation, urinary retention, nausea, vomiting, and tremors.

83.) The client has been on treatment for rheumatoid arthritis for 3 weeks. During the administration of etanercept (Enbrel), it is most important for the nurse to check:
1. The injection site for itching and edema
2. The white blood cell counts and plate

2. The white blood cell counts and platelet counts
Rationale:
Infection and pancytopenia are side effects of etanercept (Enbrel). Laboratory studies are performed before and during drug treatment. The appearance of abnormal white blood cell counts and abn

84.) Baclofen (Lioresal) is prescribed for the client with multiple sclerosis. The nurse assists in planning care, knowing that the primary therapeutic effect of this medication is which of the following?
1. Increased muscle tone
2. Decreased muscle spasm

2. Decreased muscle spasms
Rationale:
Baclofen is a skeletal muscle relaxant and central nervous system depressant and acts at the spinal cord level to decrease the frequency and amplitude of muscle spasms in clients with spinal cord injuries or diseases

85.) A nurse is monitoring a client receiving baclofen (Lioresal) for side effects related to the medication. Which of the following would indicate that the client is experiencing a side effect?
1. Polyuria
2. Diarrhea
3. Drowsiness
4. Muscular excitabili

3. Drowsiness
Rationale:
Baclofen is a central nervous system (CNS) depressant and frequently causes drowsiness, dizziness, weakness, and fatigue. It can also cause nausea, constipation, and urinary retention. Clients should be warned about the possible r

86.) A nurse is reinforcing discharge instructions to a client receiving baclofen (Lioresal). Which of the following would the nurse include in the instructions?
1. Restrict fluid intake.
2. Avoid the use of alcohol.
3. Stop the medication if diarrhea occ

2. Avoid the use of alcohol.
Rationale:
Baclofen is a central nervous system (CNS) depressant. The client should be cautioned against the use of alcohol and other CNS depressants, because baclofen potentiates the depressant activity of these agents. Const

87.) A client with acute muscle spasms has been taking baclofen (Lioresal). The client calls the clinic nurse because of continuous feelings of weakness and fatigue and asks the nurse about discontinuing the medication. The nurse should make which appropr

4. "Weakness and fatigue commonly occur and will diminish with continued medication use."
Rationale:
The client should be instructed that symptoms such as drowsiness, weakness, and fatigue are more intense in the early phase of therapy and diminish with c

88.) Dantrolene sodium (Dantrium) is prescribed for a client experiencing flexor spasms, and the client asks the nurse about the action of the medication. The nurse responds, knowing that the therapeutic action of this medication is which of the following

2. Acts directly on the skeletal muscle to relieve spasticity
Rationale:
Dantrium acts directly on skeletal muscle to relieve muscle spasticity. The primary action is the suppression of calcium release from the sarcoplasmic reticulum. This in turn decreas

89.) A nurse is reviewing the laboratory studies on a client receiving dantrolene sodium (Dantrium). Which laboratory test would identify an adverse effect associated with the administration of this medication?
1. Creatinine
2. Liver function tests
3. Blo

2. Liver function tests
Rationale:
Dose-related liver damage is the most serious adverse effect of dantrolene. To reduce the risk of liver damage, liver function tests should be performed before treatment and periodically throughout the treatment course.

90.) A nurse is reviewing the record of a client who has been prescribed baclofen (Lioresal). Which of the following disorders, if noted in the client's history, would alert the nurse to contact the health care provider?
1. Seizure disorders
2. Hyperthyro

1. Seizure disorders
Rationale:
Clients with seizure disorders may have a lowered seizure threshold when baclofen is administered. Concurrent therapy may require an increase in the anticonvulsive medication. The disorders in options 2, 3, and 4 are not a

91.) Cyclobenzaprine (Flexeril) is prescribed for a client to treat muscle spasms, and the nurse is reviewing the client's record. Which of the following disorders, if noted in the client's record, would indicate a need to contact the health care provider

1. Glaucoma
Rationale:
Because this medication has anticholinergic effects, it should be used with caution in clients with a history of urinary retention, angle-closure glaucoma, and increased intraocular pressure. Cyclobenzaprine hydrochloride should be

92.) In monitoring a client's response to disease-modifying antirheumatic drugs (DMARDs), which findings would the nurse interpret as acceptable responses? Select all that apply.
1. Symptom control during periods of emotional stress
2. Normal white blood

1. Symptom control during periods of emotional stress
2. Normal white blood cell counts, platelet, and neutrophil counts
3. Radiological findings that show nonprogression of joint degeneration
4. An increased range of motion in the affected joints 3 month

93.) The client who is human immunodeficiency virus seropositive has been taking stavudine (d4t, Zerit). The nurse monitors which of the following most closely while the client is taking this medication?
1. Gait
2. Appetite
3. Level of consciousness
4. He

1. Gait
Rationale:
Stavudine (d4t, Zerit) is an antiretroviral used to manage human immunodeficiency virus infection in clients who do not respond to or who cannot tolerate conventional therapy. The medication can cause peripheral neuropathy, and the nurs

94.) The client with acquired immunodeficiency syndrome has begun therapy with zidovudine (Retrovir, Azidothymidine, AZT, ZDV). The nurse carefully monitors which of the following laboratory results during treatment with this medication?
1. Blood culture

4. Complete blood count
Rationale:
A common side effect of therapy with zidovudine is leukopenia and anemia. The nurse monitors the complete blood count results for these changes. Options 1, 2, and 3 are unrelated to the use of this medication.

95.) The nurse is reviewing the results of serum laboratory studies drawn on a client with acquired immunodeficiency syndrome who is receiving didanosine (Videx). The nurse interprets that the client may have the medication discontinued by the health care

3. Serum amylase
Rationale:
Didanosine (Videx) can cause pancreatitis. A serum amylase level that is increased 1.5 to 2 times normal may signify pancreatitis in the client with acquired immunodeficiency syndrome and is potentially fatal. The medication ma

96.) The nurse is caring for a postrenal transplant client taking cyclosporine (Sandimmune, Gengraf, Neoral). The nurse notes an increase in one of the client's vital signs, and the client is complaining of a headache. What is the vital sign that is most

3. Blood pressure
Rationale:
Hypertension can occur in a client taking cyclosporine (Sandimmune, Gengraf, Neoral), and because this client is also complaining of a headache, the blood pressure is the vital sign to be monitoring most closely. Other adverse

97.) Amikacin (Amikin) is prescribed for a client with a bacterial infection. The client is instructed to contact the health care provider (HCP) immediately if which of the following occurs?
1. Nausea
2. Lethargy
3. Hearing loss
4. Muscle aches

3. Hearing loss
Rationale:
Amikacin (Amikin) is an aminoglycoside. Adverse effects of aminoglycosides include ototoxicity (hearing problems), confusion, disorientation, gastrointestinal irritation, palpitations, blood pressure changes, nephrotoxicity, and

98.) The nurse is assigned to care for a client with cytomegalovirus retinitis and acquired immunodeficiency syndrome who is receiving foscarnet. The nurse should check the latest results of which of the following laboratory studies while the client is ta

3. Serum creatinine
Rationale:
Foscarnet is toxic to the kidneys. Serum creatinine is monitored before therapy, two to three times per week during induction therapy, and at least weekly during maintenance therapy. Foscarnet may also cause decreased levels

99.) The client with acquired immunodeficiency syndrome and Pneumocystis jiroveci infection has been receiving pentamidine isethionate (Pentam 300). The client develops a temperature of 101� F. The nurse does further monitoring of the client, knowing that

4. The result of another infection caused by leukopenic effects of the medication.
Rationale:
Frequent side effects of this medication include leukopenia, thrombocytopenia, and anemia. The client should be monitored routinely for signs and symptoms of inf

100.) Saquinavir (Invirase) is prescribed for the client who is human immunodeficiency virus seropositive. The nurse reinforces medication instructions and tells the client to:
1. Avoid sun exposure.
2. Eat low-calorie foods.
3. Eat foods that are low in

1. Avoid sun exposure.
Rationale:
Saquinavir (Invirase) is an antiretroviral (protease inhibitor) used with other antiretroviral medications to manage human immunodeficiency virus infection. Saquinavir is administered with meals and is best absorbed if th

101.) Ketoconazole is prescribed for a client with a diagnosis of candidiasis. Select the interventions that the nurse includes when administering this medication. Select all that apply.
1. Restrict fluid intake.
2. Instruct the client to avoid alcohol.
3

2. Instruct the client to avoid alcohol.
3. Monitor hepatic and liver function studies.
5. Instruct the client to avoid exposure to the sun.
Rationale:
Ketoconazole is an antifungal medication. It is administered with food (not on an empty stomach) and an

102.) A client with human immunodeficiency virus is taking nevirapine (Viramune). The nurse should monitor for which adverse effects of the medication? Select all that apply.
1. Rash
2. Hepatotoxicity
3. Hyperglycemia
4. Peripheral neuropathy
5. Reduced b

1. Rash
2. Hepatotoxicity
Rationale:
Nevirapine (Viramune) is a non-nucleoside reverse transcriptase inhibitors (NRTI) that is used to treat HIV infection. It is used in combination with other antiretroviral medications to treat HIV. Adverse effects inclu

103.) A nurse is caring for a hospitalized client who has been taking clozapine (Clozaril) for the treatment of a schizophrenic disorder. Which laboratory study prescribed for the client will the nurse specifically review to monitor for an adverse effect

3. White blood cell count
Rationale:
Hematological reactions can occur in the client taking clozapine and include agranulocytosis and mild leukopenia. The white blood cell count should be checked before initiating treatment and should be monitored closely

104.) Disulfiram (Antabuse) is prescribed for a client who is seen in the psychiatric health care clinic. The nurse is collecting data on the client and is providing instructions regarding the use of this medication. Which is most important for the nurse

4. When the last alcoholic drink was consumed
Rationale:
Disulfiram is used as an adjunct treatment for selected clients with chronic alcoholism who want to remain in a state of enforced sobriety. Clients must abstain from alcohol intake for at least 12 h

105.) A nurse is collecting data from a client and the client's spouse reports that the client is taking donepezil hydrochloride (Aricept). Which disorder would the nurse suspect that this client may have based on the use of this medication?
1. Dementia
2

1. Dementia
Rationale:
Donepezil hydrochloride is a cholinergic agent used in the treatment of mild to moderate dementia of the Alzheimer type. It enhances cholinergic functions by increasing the concentration of acetylcholine. It slows the progression of

106.) Fluoxetine (Prozac) is prescribed for the client. The nurse reinforces instructions to the client regarding the administration of the medication. Which statement by the client indicates an understanding about administration of the medication?
1. "I

3. "I should take the medication in the morning when I first arise."
Rationale:
Fluoxetine hydrochloride is administered in the early morning without consideration to meals.
*
Eliminate options 1, 2, and 4 because they are comparable or alike and indicate

107.) A client receiving a tricyclic antidepressant arrives at the mental health clinic. Which observation indicates that the client is correctly following the medication plan?
1. Reports not going to work for this past week
2. Complains of not being able

3. Arrives at the clinic neat and appropriate in appearance
Rationale:
Depressed individuals will sleep for long periods, are not able to go to work, and feel as if they cannot "do anything." Once they have had some therapeutic effect from their medicatio

108.) A nurse is performing a follow-up teaching session with a client discharged 1 month ago who is taking fluoxetine (Prozac). What information would be important for the nurse to gather regarding the adverse effects related to the medication?
1. Cardio

2. Gastrointestinal dysfunctions
Rationale:
The most common adverse effects related to fluoxetine include central nervous system (CNS) and gastrointestinal (GI) system dysfunction. This medication affects the GI system by causing nausea and vomiting, cram

109.) A client taking buspirone (BuSpar) for 1 month returns to the clinic for a follow-up visit. Which of the following would indicate medication effectiveness?
1. No rapid heartbeats or anxiety
2. No paranoid thought processes
3. No thought broadcasting

1. No rapid heartbeats or anxiety
Rationale:
Buspirone hydrochloride is not recommended for the treatment of drug or alcohol withdrawal, paranoid thought disorders, or schizophrenia (thought broadcasting or delusions). Buspirone hydrochloride is most ofte

110.) A client taking lithium carbonate (Lithobid) reports vomiting, abdominal pain, diarrhea, blurred vision, tinnitus, and tremors. The lithium level is checked as a part of the routine follow-up and the level is 3.0 mEq/L. The nurse knows that this lev

1. Toxic
Rationale:
The therapeutic serum level of lithium is 0.6 to 1.2 mEq/L. A level of 3 mEq/L indicates toxicity.

111.) A client arrives at the health care clinic and tells the nurse that he has been doubling his daily dosage of bupropion hydrochloride (Wellbutrin) to help him get better faster. The nurse understands that the client is now at risk for which of the fo

3. Seizure activity
Rationale:
Bupropion does not cause significant orthostatic blood pressure changes. Seizure activity is common in dosages greater than 450 mg daily. Bupropion frequently causes a drop in body weight. Insomnia is a side effect, but seiz

112.) A hospitalized client is started on phenelzine sulfate (Nardil) for the treatment of depression. The nurse instructs the client to avoid consuming which foods while taking this medication? Select all that apply.
1. Figs
2. Yogurt
3. Crackers
4. Aged

1. Figs
2. Yogurt
4. Aged cheese
Rationale:
Phenelzine sulfate (Nardil) is a monoamine oxidase inhibitor(MAOI). The client should avoid taking in foods that are high in tyramine. Use of these foods could trigger a potentially fatal hypertensive crisis. So

113.) A nurse is reinforcing discharge instructions to a client receiving sulfisoxazole. Which of the following would be included in the plan of care for instructions?
1. Maintain a high fluid intake.
2. Discontinue the medication when feeling better.
3.

1. Maintain a high fluid intake.
Rationale:
Each dose of sulfisoxazole should be administered with a full glass of water, and the client should maintain a high fluid intake. The medication is more soluble in alkaline urine. The client should not be instru

114.) A postoperative client requests medication for flatulence (gas pains). Which medication from the following PRN list should the nurse administer to this client?
1. Ondansetron (Zofran)
2. Simethicone (Mylicon)
3. Acetaminophen (Tylenol)
4. Magnesium

2. Simethicone (Mylicon)
Rationale:
Simethicone is an antiflatulent used in the relief of pain caused by excessive gas in the gastrointestinal tract. Ondansetron is used to treat postoperative nausea and vomiting. Acetaminophen is a nonopioid analgesic. M

115.) A client received 20 units of NPH insulin subcutaneously at 8:00 AM. The nurse should check the client for a potential hypoglycemic reaction at what time?
1. 5:00 PM
2. 10:00 AM
3. 11:00 AM
4. 11:00 PM

1. 5:00 PM
Rationale:
NPH is intermediate-acting insulin. Its onset of action is 1 to 2� hours, it peaks in 4 to 12 hours, and its duration of action is 24 hours. Hypoglycemic reactions most likely occur during peak time.

116.) A nurse administers a dose of scopolamine (Transderm-Scop) to a postoperative client. The nurse tells the client to expect which of the following side effects of this medication?
1. Dry mouth
2. Diaphoresis
3. Excessive urination
4. Pupillary constr

1. Dry mouth
Rationale:
Scopolamine is an anticholinergic medication for the prevention of nausea and vomiting that causes the frequent side effects of dry mouth, urinary retention, decreased sweating, and dilation of the pupils. The other options describ

117.) A nurse has given the client taking ethambutol (Myambutol) information about the medication. The nurse determines that the client understands the instructions if the client immediately reports:
1. Impaired sense of hearing
2. Distressing gastrointes

4. Difficulty discriminating the color red from green
Rationale:
Ethambutol causes optic neuritis, which decreases visual acuity and the ability to discriminate between the colors red and green. This poses a potential safety hazard when driving a motor ve

118.) A nurse is caring for an older client with a diagnosis of myasthenia gravis and has reinforced self-care instructions. Which statement by the client indicates that further teaching is necessary?
1. "I rest each afternoon after my walk."
2. "I cough

4. "I can change the time of my medication on the mornings that I feel strong."
Rationale:
The client with myasthenia gravis should be taught that timing of anticholinesterase medication is critical. It is important to instruct the client to administer th

119.) A client with diabetes mellitus who has been controlled with daily insulin has been placed on atenolol (Tenormin) for the control of angina pectoris. Because of the effects of atenolol, the nurse determines that which of the following is the most re

4. Low blood glucose level
Rationale:
?-Adrenergic blocking agents, such as atenolol, inhibit the appearance of signs and symptoms of acute hypoglycemia, which would include nervousness, increased heart rate, and sweating. Therefore, the client receiving

120.) A client is taking lansoprazole (Prevacid) for the chronic management of Zollinger-Ellison syndrome. The nurse advises the client to take which of the following products if needed for a headache?
1. Naprosyn (Aleve)
2. Ibuprofen (Advil)
3. Acetamino

3. Acetaminophen (Tylenol)
Rationale:
Zollinger-Ellison syndrome is a hypersecretory condition of the stomach. The client should avoid taking medications that are irritating to the stomach lining. Irritants would include aspirin and nonsteroidal antiinfla

121.) A client who is taking hydrochlorothiazide (HydroDIURIL, HCTZ) has been started on triamterene (Dyrenium) as well. The client asks the nurse why both medications are required. The nurse formulates a response, based on the understanding that:
1. Both

4. Triamterene is a potassium-sparing diuretic, whereas hydrochlorothiazide is a potassium-losing diuretic.
Rationale:
Potassium-sparing diuretics include amiloride (Midamor), spironolactone (Aldactone), and triamterene (Dyrenium). They are weak diuretics

122.) A client who has begun taking fosinopril (Monopril) is very distressed, telling the nurse that he cannot taste food normally since beginning the medication 2 weeks ago. The nurse provides the best support to the client by:
1. Telling the client not

3. Informing the client that impaired taste is expected and generally disappears in 2 to 3 months
Rationale:
ACE inhibitors, such as fosinopril, cause temporary impairment of taste (dysgeusia). The nurse can tell the client that this effect usually disapp

123.) A nurse is planning to administer amlodipine (Norvasc) to a client. The nurse plans to check which of the following before giving the medication?
1. Respiratory rate
2. Blood pressure and heart rate
3. Heart rate and respiratory rate
4. Level of con

2. Blood pressure and heart rate
Rationale:
Amlodipine is a calcium channel blocker. This medication decreases the rate and force of cardiac contraction. Before administering a calcium channel blocking agent, the nurse should check the blood pressure and

124.) A client with chronic renal failure is receiving ferrous sulfate (Feosol). The nurse monitors the client for which common side effect associated with this medication?
1. Diarrhea
2. Weakness
3. Headache
4. Constipation

4. Constipation
Rationale:
Feosol is an iron supplement used to treat anemia. Constipation is a frequent and uncomfortable side effect associated with the administration of oral iron supplements. Stool softeners are often prescribed to prevent constipatio

125.) A nurse is preparing to administer digoxin (Lanoxin), 0.125 mg orally, to a client with heart failure. Which vital sign is most important for the nurse to check before administering the medication?
1. Heart rate
2. Temperature
3. Respirations
4. Blo

1. Heart rate
Rationale:
Digoxin is a cardiac glycoside that is used to treat heart failure and acts by increasing the force of myocardial contraction. Because bradycardia may be a clinical sign of toxicity, the nurse counts the apical heart rate for 1 fu

126.) A nurse is caring for a client who has been prescribed furosemide (Lasix) and is monitoring for adverse effects associated with this medication. Which of the following should the nurse recognize as a potential adverse effect Select all that apply.
1

2. Tinnitus
3. Hypotension
4. Hypokalemia
Rationale:
Furosemide is a loop diuretic; therefore, an expected effect is increased urinary frequency. Nausea is a frequent side effect, not an adverse effect. Photosensitivity is an occasional side effect. Adver

127.) The nurse provides medication instructions to an older hypertensive client who is taking 20 mg of lisinopril (Prinivil, Zestril) orally daily. The nurse evaluates the need for further teaching when the client states which of the following?
1. "I can

1. "I can skip a dose once a week."
Rationale:
Lisinopril is an antihypertensive angiotensin-converting enzyme (ACE) inhibitor. The usual dosage range is 20 to 40 mg per day. Adverse effects include headache, dizziness, fatigue, orthostatic hypotension, t

128.) A nurse is providing instructions to an adolescent who has a history of seizures and is taking an anticonvulsant medication. Which of the following statements indicates that the client understands the instructions?
1. "I will never be able to drive

3. "I can't drink alcohol while I am taking my medication."
Rationale:
Alcohol will lower the seizure threshold and should be avoided. Adolescents can obtain a driver's license in most states when they have been seizure free for 1 year. Anticonvulsants ca

129.) Megestrol acetate (Megace), an antineoplastic medication, is prescribed for the client with metastatic endometrial carcinoma. The nurse reviews the client's history and contacts the registered nurse if which diagnosis is documented in the client's h

3. Thrombophlebitis
Rationale:
Megestrol acetate (Megace) suppresses the release of luteinizing hormone from the anterior pituitary by inhibiting pituitary function and regressing tumor size. Megestrol is used with caution if the client has a history of t

130.) The nurse is analyzing the laboratory results of a client with leukemia who has received a regimen of chemotherapy. Which laboratory value would the nurse specifically note as a result of the massive cell destruction that occurred from the chemother

3. Increased uric acid level
Rationale:
Hyperuricemia is especially common following treatment for leukemias and lymphomas because chemotherapy results in a massive cell kill. Although options 1, 2, and 4 also may be noted, an increased uric acid level is

131.) The nurse is reinforcing medication instructions to a client with breast cancer who is receiving cyclophosphamide (Neosar). The nurse tells the client to:
1. Take the medication with food.
2. Increase fluid intake to 2000 to 3000 mL daily.
3. Decrea

2. Increase fluid intake to 2000 to 3000 mL daily.
Rationale:
Hemorrhagic cystitis is a toxic effect that can occur with the use of cyclophosphamide. The client needs to be instructed to drink copious amounts of fluid during the administration of this med

132.) The client with non-Hodgkin's lymphoma is receiving daunorubicin (DaunoXome). Which of the following would indicate to the nurse that the client is experiencing a toxic effect related to the medication?
1. Fever
2. Diarrhea
3. Complaints of nausea a

4. Crackles on auscultation of the lungs
Rationale:
Cardiotoxicity noted by abnormal electrocardiographic findings or cardiomyopathy manifested as congestive heart failure is a toxic effect of daunorubicin. Bone marrow depression is also a toxic effect. N

133.) A nurse is monitoring a client receiving desmopressin acetate (DDAVP) for adverse effects to the medication. Which of the following indicates the presence of an adverse effect?
1. Insomnia
2. Drowsiness
3. Weight loss
4. Increased urination

2. Drowsiness
Rationale:
Water intoxication (overhydration) or hyponatremia is an adverse effect to desmopressin. Early signs include drowsiness, listlessness, and headache. Decreased urination, rapid weight gain, confusion, seizures, and coma also may oc

134.) A nurse reinforces instructions to a client who is taking levothyroxine (Synthroid). The nurse tells the client to take the medication:
1. With food
2. At lunchtime
3. On an empty stomach
4. At bedtime with a snack

Rationale:
Oral doses of levothyroxine (Synthroid) should be taken on an empty stomach to enhance absorption. Dosing should be done in the morning before breakfast.
*
Note that options 1, 2, and 4 are comparable or alike in that these options address admi

135.) A nurse reinforces medication instructions to a client who is taking levothyroxine (Synthroid). The nurse instructs the client to notify the health care provider (HCP) if which of the following occurs?
1. Fatigue
2. Tremors
3. Cold intolerance
4. Ex

2. Tremors
Rationale:
Excessive doses of levothyroxine (Synthroid) can produce signs and symptoms of hyperthyroidism. These include tachycardia, chest pain, tremors, nervousness, insomnia, hyperthermia, heat intolerance, and sweating. The client should be

136.) A nurse performs an admission assessment on a client who visits a health care clinic for the first time. The client tells the nurse that propylthiouracil (PTU) is taken daily. The nurse continues to collect data from the client, suspecting that the

2. Graves' disease
Rationale:
PTU inhibits thyroid hormone synthesis and is used to treat hyperthyroidism, or Graves' disease. Myxedema indicates hypothyroidism.
Cushing's syndrome and Addison's disease are disorders related to adrenal function.

137.) A nurse is reinforcing instructions for a client regarding intranasal desmopressin acetate (DDAVP). The nurse tells the client that which of the following is a side effect of the medication?
1. Headache
2. Vulval pain
3. Runny nose
4. Flushed skin

3. Runny nose
Rationale:
Desmopressin administered by the intranasal route can cause a runny or stuffy nose. Headache, vulval pain, and flushed skin are side effects if the medication is administered by the intravenous (IV) route.

138.) A daily dose of prednisone is prescribed for a client. A nurse reinforces instructions to the client regarding administration of the medication and instructs the client that the best time to take this medication is:
1. At noon
2. At bedtime
3. Early

3. Early morning
Rationale:
Corticosteroids (glucocorticoids) should be administered before 9:00 AM. Administration at this time helps minimize adrenal insufficiency and mimics the burst of glucocorticoids released naturally by the adrenal glands each mor

139.) Prednisone is prescribed for a client with diabetes mellitus who is taking Humulin neutral protamine Hagedorn (NPH) insulin daily. Which of the following prescription changes does the nurse anticipate during therapy with the prednisone?
1. An additi

3. An increased amount of daily Humulin NPH insulin
Rationale:
Glucocorticoids can elevate blood glucose levels. Clients with diabetes mellitus may need their dosages of insulin or oral hypoglycemic medications increased during glucocorticoid therapy. The

140.) The client has a new prescription for metoclopramide (Reglan). On review of the chart, the nurse identifies that this medication can be safely administered with which condition?
1. Intestinal obstruction
2. Peptic ulcer with melena
3. Diverticulitis

4. Vomiting following cancer chemotherapy
Rationale:
Metoclopramide is a gastrointestinal (GI) stimulant and antiemetic. Because it is a GI stimulant, it is contraindicated with GI obstruction, hemorrhage, or perforation. It is used in the treatment of em

141.) The nurse has reinforced instructions to a client who has been prescribed cholestyramine (Questran). Which statement by the client indicates a need for further instructions?
1. "I will continue taking vitamin supplements."
2. "This medication will h

3. "This medication should only be taken with water."
Rationale:
Cholestyramine (Questran) is a bile acid sequestrant used to lower the cholesterol level, and client compliance is a problem because of its taste and palatability. The use of flavored produc

142.) A health care provider has written a prescription for ranitidine (Zantac), once daily. The nurse should schedule the medication for which of the following times?
1. At bedtime
2. After lunch
3. With supper
4. Before breakfast

1. At bedtime
Rationale:
A single daily dose of ranitidine is usually scheduled to be given at bedtime. This allows for a prolonged effect, and the greatest protection of the gastric mucosa.
*
recall that ranitidine suppresses secretions of gastric acids

143.) A client has just taken a dose of trimethobenzamide (Tigan). The nurse plans to monitor this client for relief of:
1. Heartburn
2. Constipation
3. Abdominal pain
4. Nausea and vomiting

4. Nausea and vomiting
Rationale:
Trimethobenzamide is an antiemetic agent used in the treatment of nausea and vomiting. The other options are incorrect.

144.) A client is taking docusate sodium (Colace). The nurse monitors which of the following to determine whether the client is having a therapeutic effect from this medication?
1. Abdominal pain
2. Reduction in steatorrhea
3. Hematest-negative stools
4.

4. Regular bowel movements
Rationale:
Docusate sodium is a stool softener that promotes the absorption of water into the stool, producing a softer consistency of stool. The intended effect is relief or prevention of constipation. The medication does not r

145.) A nurse has a prescription to give a client albuterol (Proventil HFA) (two puffs) and beclomethasone dipropionate (Qvar) (nasal inhalation, two puffs), by metered-dose inhaler. The nurse administers the medication by giving the:
1. Albuterol first a

1. Albuterol first and then the beclomethasone dipropionate
Rationale:
Albuterol is a bronchodilator. Beclomethasone dipropionate is a glucocorticoid. Bronchodilators are always administered before glucocorticoids when both are to be given on the same tim

146.) A client has begun therapy with theophylline (Theo-24). The nurse tells the client to limit the intake of which of the following while taking this medication?
1. Oranges and pineapple
2. Coffee, cola, and chocolate
3. Oysters, lobster, and shrimp
4.

2. Coffee, cola, and chocolate
Rationale:
Theophylline is a xanthine bronchodilator. The nurse teaches the client to limit the intake of xanthine-containing foods while taking this medication. These include coffee, cola, and chocolate.

147.) A client with a prescription to take theophylline (Theo-24) daily has been given medication instructions by the nurse. The nurse determines that the client needs further information about the medication if the client states that he or she will:
1. D

2. Take the daily dose at bedtime.
Rationale:
The client taking a single daily dose of theophylline, a xanthine bronchodilator, should take the medication early in the morning. This enables the client to have maximal benefit from the medication during day

148.) A client is taking cetirizine hydrochloride (Zyrtec). The nurse checks for which of the following side effects of this medication?
1. Diarrhea
2. Excitability
3. Drowsiness
4. Excess salivation

3. Drowsiness
Rationale:
A frequent side effect of cetirizine hydrochloride (Zyrtec), an antihistamine, is drowsiness or sedation. Others include blurred vision, hypertension (and sometimes hypotension), dry mouth, constipation, urinary retention, and swe

149.) A client taking fexofenadine (Allegra) is scheduled for allergy skin testing and tells the nurse in the health care provider's office that a dose was taken this morning. The nurse determines that:
1. The client should reschedule the appointment.
2.

1. The client should reschedule the appointment.
Rationale:
Fexofenadine is an antihistamine, which provides relief of symptoms caused by allergy. Antihistamines should be discontinued for at least 3 days (72 hours) before allergy skin testing to avoid fa

150.) A client complaining of not feeling well is seen in a clinic. The client is taking several medications for the control of heart disease and hypertension. These medications include a ?-blocker, digoxin (Lanoxin), and a diuretic. A tentative diagnosis

3. Double vision, loss of appetite, and nausea
Rationale:
Double vision, loss of appetite, and nausea are signs of digoxin toxicity. Additional signs of digoxin toxicity include bradycardia, difficulty reading, visual alterations such as green and yellow

151.) A client is being treated for acute congestive heart failure with intravenously administered bumetanide. The vital signs are as follows: blood pressure, 100/60 mm Hg; pulse, 96 beats/min; and respirations, 24 breaths/min. After the initial dose, whi

3. Monitoring blood pressure
Rationale:
Bumetanide is a loop diuretic. Hypotension is a common side effect associated with the use of this medication. The other options also require assessment but are not the priority.
*
priority ABCs�airway, breathing, a

152.) Intravenous heparin therapy is prescribed for a client. While implementing this prescription, a nurse ensures that which of the following medications is available on the nursing unit?
1. Protamine sulfate
2. Potassium chloride
3. Phytonadione (vitam

1. Protamine sulfate
Rationale:
The antidote to heparin is protamine sulfate; it should be readily available for use if excessive bleeding or hemorrhage occurs. Potassium chloride is administered for a potassium deficit. Vitamin K is an antidote for warfa

153.) A client is diagnosed with pulmonary embolism and is to be treated with streptokinase (Streptase). A nurse would report which priority data collection finding to the registered nurse before initiating this therapy?
1. Adventitious breath sounds
2. T

3. Blood pressure of 198/110 mm Hg
Rationale:
Thrombolytic therapy is contraindicated in a number of preexisting conditions in which there is a risk of uncontrolled bleeding, similar to the case in anticoagulant therapy. Thrombolytic therapy also is contr

154.) A nurse is reinforcing dietary instructions to a client who has been prescribed cyclosporine (Sandimmune). Which food item would the nurse instruct the client to avoid?
1. Red meats
2. Orange juice
3. Grapefruit juice
4. Green, leafy vegetables

3. Grapefruit juice
Rationale:
A compound present in grapefruit juice inhibits metabolism of cyclosporine. As a result, the consumption of grapefruit juice can raise cyclosporine levels by 50% to 100%, thereby greatly increasing the risk of toxicity. Grap

155.) Mycophenolate mofetil (CellCept) is prescribed for a client as prophylaxis for organ rejection following an allogeneic renal transplant. Which of the following instructions does the nurse reinforce regarding administration of this medication?
1. Adm

4. Contact the health care provider (HCP) if a sore throat occurs.
Rationale:
Mycophenolate mofetil should be administered on an empty stomach. The capsules should not be opened or crushed. The client should contact the HCP if unusual bleeding or bruising

156.) A nurse is reviewing the laboratory results for a client receiving tacrolimus (Prograf). Which laboratory result would indicate to the nurse that the client is experiencing an adverse effect of the medication?
1. Blood glucose of 200 mg/dL
2. Potass

1. Blood glucose of 200 mg/dL
Rationale:
A blood glucose level of 200 mg/dL is elevated above the normal range of 70 to 110 mg/dL and suggests an adverse effect. Other adverse effects include neurotoxicity evidenced by headache, tremor, insomnia; gastroin

157.) A client receiving nitrofurantoin (Macrodantin) calls the health care provider's office complaining of side effects related to the medication. Which side effect indicates the need to stop treatment with this medication?
1. Nausea
2. Diarrhea
3. Anor

4. Cough and chest pain
Rationale:
Gastrointestinal (GI) effects are the most frequent adverse reactions to this medication and can be minimized by administering the medication with milk or meals. Pulmonary reactions, manifested as dyspnea, chest pain, ch

158.) A client with chronic renal failure is receiving epoetin alfa (Epogen, Procrit). Which laboratory result would indicate a therapeutic effect of the medication?
1. Hematocrit of 32%
2. Platelet count of 400,000 cells/mm3
3. White blood cell count of

1. Hematocrit of 32%
Rationale:
Epoetin alfa is used to reverse anemia associated with chronic renal failure. A therapeutic effect is seen when the hematocrit is between 30% and 33%. The laboratory tests noted in the other options are unrelated to the use

159.) A nurse is caring for a client receiving morphine sulfate subcutaneously for pain. Because morphine sulfate has been prescribed for this client, which nursing action would be included in the plan of care?
1. Encourage fluid intake.
2. Monitor the cl

4. Encourage the client to cough and deep breathe.
Rationale:
Morphine sulfate suppresses the cough reflex. Clients need to be encouraged to cough and deep breathe to prevent pneumonia.
*
ABCs�airway, breathing, and circulation
n**

160.) Meperidine hydrochloride (Demerol) is prescribed for the client with pain. Which of the following would the nurse monitor for as a side effect of this medication?
1. Diarrhea
2. Bradycardia
3. Hypertension
4. Urinary retention

4. Urinary retention
Rationale:
Meperidine hydrochloride (Demerol) is an opioid analgesic. Side effects of this medication include respiratory depression, orthostatic hypotension, tachycardia, drowsiness and mental clouding, constipation, and urinary rete

161.) A nurse is caring for a client with severe back pain, and codeine sulfate has been prescribed for the client. Which of the following would the nurse include in the plan of care while the client is taking this medication?
1. Restrict fluid intake.
2.

2. Monitor bowel activity.
Rationale:
While the client is taking codeine sulfate, an opioid analgesic, the nurse would monitor vital signs and monitor for hypotension. The nurse should also increase fluid intake, palpate the bladder for urinary retention,

162.) Carbamazepine (Tegretol) is prescribed for a client with a diagnosis of psychomotor seizures. The nurse reviews the client's health history, knowing that this medication is contraindicated if which of the following disorders is present?
1. Headaches

2. Liver disease
Rationale:
Carbamazepine (Tegretol) is contraindicated in liver disease, and liver function tests are routinely prescribed for baseline purposes and are monitored during therapy. It is also contraindicated if the client has a history of b

163.) A client with trigeminal neuralgia tells the nurse that acetaminophen (Tylenol) is taken on a frequent daily basis for relief of generalized discomfort. The nurse reviews the client's laboratory results and determines that which of the following ind

4. A direct bilirubin level of 2 mg/dL
Rationale:
In adults, overdose of acetaminophen (Tylenol) causes liver damage. Option 4 is an indicator of liver function and is the only option that indicates an abnormal laboratory value. The normal direct bilirubi

164.) A client receives a prescription for methocarbamol (Robaxin), and the nurse reinforces instructions to the client regarding the medication. Which client statement would indicate a need for further instructions?
1. "My urine may turn brown or green.

3. "If my vision becomes blurred, I don't need to be concerned about it."
Rationale:
The client needs to be told that the urine may turn brown, black, or green. Other adverse effects include blurred vision, nasal congestion, urticaria, and rash. The clien

165.) The client has been on treatment for rheumatoid arthritis for 3 weeks. During the administration of etanercept (Enbrel), it is most important for the nurse to assess:
1. The injection site for itching and edema
2. The white blood cell counts and pla

2. The white blood cell counts and platelet counts
Rationale:
Infection and pancytopenia are adverse effects of etanercept (Enbrel). Laboratory studies are performed before and during treatment. The appearance of abnormal white blood cell counts and abnor

166.) Alendronate (Fosamax) is prescribed for a client with osteoporosis. The client taking this medication is instructed to:
1. Take the medication at bedtime.
2. Take the medication in the morning with breakfast.
3. Lie down for 30 minutes after taking

4. Take the medication with a full glass of water after rising in the morning.
Rationale:
Precautions need to be taken with administration of alendronate to prevent gastrointestinal side effects (especially esophageal irritation) and to increase absorptio

167.) A nurse prepares to reinforce instructions to a client who is taking allopurinol (Zyloprim). The nurse plans to include which of the following in the instructions?
1. Instruct the client to drink 3000 mL of fluid per day.
2. Instruct the client to t

1. Instruct the client to drink 3000 mL of fluid per day.
Rationale:
Allopurinol (Zyloprim) is an antigout medication used to decrease uric acid levels. Clients taking allopurinol are encouraged to drink 3000 mL of fluid a day. A full therapeutic effect m

168.) Colcrys (colchicine) is prescribed for a client with a diagnosis of gout. The nurse reviews the client's medical history in the health record, knowing that the medication would be contraindicated in which disorder?
1. Myxedema
2. Renal failure
3. Hy

2. Renal failure
Rationale:
Colchicine is contraindicated in clients with severe gastrointestinal, renal, hepatic or cardiac disorders, or with blood dyscrasias. Clients with impaired renal function may exhibit myopathy and neuropathy manifested as genera

169.) Insulin glargine (Lantus) is prescribed for a client with diabetes mellitus. The nurse tells the client that it is best to take the insulin:
1. 1 hour after each meal
2. Once daily, at the same time each day
3. 15 minutes before breakfast, lunch, an

2. Once daily, at the same time each day
Rationale:
Insulin glargine is a long-acting recombinant DNA human insulin used to treat type 1 and type 2 diabetes mellitus. It has a 24-hour duration of action and is administered once a day, at the same time eac

170.) Atenolol hydrochloride (Tenormin) is prescribed for a hospitalized client. The nurse should perform which of the following as a priority action before administering the medication?
1. Listen to the client's lung sounds.
2. Check the client's blood p

2. Check the client's blood pressure.
Rationale:
Atenolol hydrochloride is a beta-blocker used to treat hypertension. Therefore the priority nursing action before administration of the medication is to check the client's blood pressure. The nurse also che

171.) A nurse is preparing to administer furosemide (Lasix) to a client with a diagnosis of heart failure. The most important laboratory test result for the nurse to check before administering this medication is:
1. Potassium level
2. Creatinine level
3.

1. Potassium level
Rationale:
Furosemide is a loop diuretic. The medication causes a decrease in the client's electrolytes, especially potassium, sodium, and chloride. Administering furosemide to a client with low electrolyte levels could precipitate vent

172.) A nurse provides dietary instructions to a client who will be taking warfarin sodium (Coumadin). The nurse tells the client to avoid which food item?
1. Grapes
2. Spinach
3. Watermelon
4. Cottage cheese

2. Spinach
Rationale:
Warfarin sodium is an anticoagulant. Anticoagulant medications act by antagonizing the action of vitamin K, which is needed for clotting. When a client is taking an anticoagulant, foods high in vitamin K often are omitted from the di

173.) A nurse reviews the medication history of a client admitted to the hospital and notes that the client is taking leflunomide (Arava). During data collection, the nurse asks which question to determine medication effectiveness?
1. "Do you have any joi

1. "Do you have any joint pain?"
Rationale:
Leflunomide is an immunosuppressive agent and has an anti-inflammatory action. The medication provides symptomatic relief of rheumatoid arthritis. Diarrhea can occur as a side effect of the medication. The other

174.) A client with portosystemic encephalopathy is receiving oral lactulose (Chronulac) daily. The nurse assesses which of the following to determine medication effectiveness?
1. Lung sounds
2. Blood pressure
3. Blood ammonia level
4. Serum potassium lev

3. Blood ammonia level
Rationale:
Lactulose is a hyperosmotic laxative and ammonia detoxicant. It is used to prevent or treat portosystemic encephalopathy, including hepatic precoma and coma. It also is used to treat constipation. The medication retains a

175.) A nurse notes that a client is receiving lamivudine (Epivir). The nurse determines that this medication has been prescribed to treat which of the following?
1. Pancreatitis
2. Pharyngitis
3. Tonic-clonic seizures
4. Human immunodeficiency virus (HIV

4. Human immunodeficiency virus (HIV) infection
Rationale:
Lamivudine is a nucleoside reverse transcriptase inhibitor and antiviral medication. It slows HIV replication and reduces the progression of HIV infection. It also is used to treat chronic hepatit

176.) A nurse notes that a client is taking lansoprazole (Prevacid). On data collection, the nurse asks which question to determine medication effectiveness?
1. "Has your appetite increased?"
2. "Are you experiencing any heartburn?"
3. "Do you have any pr

2. "Are you experiencing any heartburn?"
Rationale:
Lansoprazole is a gastric acid pump inhibitor used to treat gastric and duodenal ulcers, erosive esophagitis, and hypersecretory conditions. It also is used to treat gastroesophageal reflux disease (GERD

177.) A nurse is assisting in caring for a pregnant client who is receiving intravenous magnesium sulfate for the management of preeclampsia and notes that the client's deep tendon reflexes are absent. On the basis of this data, the nurse reports the find

4. The client is experiencing magnesium toxicity.
Rationale:
Magnesium toxicity can occur as a result of magnesium sulfate therapy. Signs of magnesium sulfate toxicity relate to the central nervous system depressant effects of the medication and include r

178.) Methylergonovine (Methergine) is prescribed for a client with postpartum hemorrhage caused by uterine atony. Before administering the medication, the nurse checks which of the following as the important client parameter?
1. Temperature
2. Lochial fl

4. Blood pressure
Rationale:
Methylergonovine is an ergot alkaloid used for postpartum hemorrhage. It stimulates contraction of the uterus and causes arterial vasoconstriction. Ergot alkaloids are avoided in clients with significant cardiovascular disease

179.) A nurse provides medication instructions to a client who had a kidney transplant about therapy with cyclosporine (Sandimmune). Which statement by the client indicates a need for further instruction?
1. "I need to obtain a yearly influenza vaccine.

1. "I need to obtain a yearly influenza vaccine."
Rationale:
Cyclosporine is an immunosuppressant medication. Because of the medication's effects, the client should not receive any vaccinations without first consulting the HCP. The client should report de

180.) A health care provider (HCP) writes a prescription for digoxin (Lanoxin), 0.25 mg daily. The nurse teaches the client about the medication and tells the client that it is important to:
1. Count the radial and carotid pulses every morning.
2. Check t

4. Withhold the medication and call the HCP if the pulse is less than 60 beats per minute.
Rationale:
An important component of taking this medication is monitoring the pulse rate; however, it is not necessary for the client to take both the radial and ca

181.) A client is taking ticlopidine hydrochloride (Ticlid). The nurse tells the client to avoid which of the following while taking this medication?
1. Vitamin C
2. Vitamin D
3. Acetaminophen (Tylenol)
4. Acetylsalicylic acid (aspirin)

4. Acetylsalicylic acid (aspirin)
Rationale:
Ticlopidine hydrochloride is a platelet aggregation inhibitor. It is used to decrease the risk of thrombotic strokes in clients with precursor symptoms. Because it is an antiplatelet agent, other medications th

182.) A client with angina pectoris is experiencing chest pain that radiates down the left arm. The nurse administers a sublingual nitroglycerin tablet to the client. The client's pain is unrelieved, and the nurse determines that the client needs another

Rationale:
Nitroglycerin acts directly on the smooth muscle of the blood vessels, causing relaxation and dilation. As a result, hypotension can occur. The nurse would check the client's blood pressure before administering the second nitroglycerin tablet.

183.) A client who received a kidney transplant is taking azathioprine (Imuran), and the nurse provides instructions about the medication. Which statement by the client indicates a need for further instructions?
1. "I need to watch for signs of infection.

2. "I need to discontinue the medication after 14 days of use."
Rationale:
Azathioprine is an immunosuppressant medication that is taken for life. Because of the effects of the medication, the client must watch for signs of infection, which are reported i

184.) A nurse preparing a client for surgery reviews the client's medication record. The client is to be nothing per mouth (NPO) after midnight. Which of the following medications, if noted on the client's record, should the nurse question?
1. Cyclobenzap

4. Prednisone
Rationale:
Prednisone is a corticosteroid that can cause adrenal atrophy, which reduces the body's ability to withstand stress. Before and during surgery, dosages may be temporarily increased. Cyclobenzaprine is a skeletal muscle relaxant. A

185.) Which of the following herbal therapies would be prescribed for its use as an antispasmodic? Select all that apply.
1.Aloe
2.Kava
3.Ginger
4.Chamomile
5.Peppermint oil

4.Chamomile
5.Peppermint oil
Rationale:
Chamomile has a mild sedative effect and acts as an antispasmodic and anti-inflammatory. Peppermint oil acts as an antispasmodic and is used for irritable bowel syndrome. Topical aloe promotes wound healing. Aloe ta

186.) A nurse prepares to administer sodium polystyrene sulfonate (Kayexalate) to a client. Before administering the medication, the nurse reviews the action of the medication and understands that it:
1. Releases bicarbonate in exchange for primarily sodi

2. Releases sodium ions in exchange for primarily potassium ions
Rationale:
Sodium polystyrene sulfonate is a cation exchange resin used in the treatment of hyperkalemia. The resin either passes through the intestine or is retained in the colon. It releas

187.) A clinic nurse prepares to administer an MMR (measles, mumps, rubella) vaccine to a child. How is this vaccine best administered?
1. Intramuscularly in the deltoid muscle
2. Subcutaneously in the gluteal muscle
3. Subcutaneously in the outer aspect

3. Subcutaneously in the outer aspect of the upper arm
Rationale:
The MMR vaccine is administered subcutaneously in the outer aspect of the upper arm. The gluteal muscle is most often used for intramuscular injections. The MMR vaccine is not administered

188.) The nurse should anticipate that the most likely medication to be prescribed prophylactically for a child with spina bifida (myelomeningocele) who has a neurogenic bladder would be:
1. Prednisone
2. Sulfisoxazole
3. Furosemide (Lasix)
4. Intravenous

2. Sulfisoxazole
Rationale:
A neurogenic bladder prevents the bladder from completely emptying because of the decrease in muscle tone. The most likely medication to be prescribed to prevent urinary tract infection would be an antibiotic. A common prescrib

189.) Prostaglandin E1 is prescribed for a child with transposition of the great arteries. The mother of the child asks the nurse why the child needs the medication. The nurse tells the mother that the medication:
1. Prevents hypercyanotic (blue or tet) s

4. Provides adequate oxygen saturation and maintains cardiac output
Rationale:
A child with transposition of the great arteries may receive prostaglandin E1 temporarily to increase blood mixing if systemic and pulmonary mixing are inadequate to maintain a

190.) A child is hospitalized with a diagnosis of lead poisoning. The nurse assisting in caring for the child would prepare to assist in administering which of the following medications?
1. Activated charcoal
2. Sodium bicarbonate
3. Syrup of ipecac syrup

4. Dimercaprol (BAL in Oil)
Rationale:
Dimercaprol is a chelating agent that is administered to remove lead from the circulating blood and from some tissues and organs for excretion in the urine. Sodium bicarbonate may be used in salicylate poisoning. Syr

191.) A child is brought to the emergency department for treatment of an acute asthma attack. The nurse prepares to administer which of the following medications first?
1. Oral corticosteroids
2. A leukotriene modifier
3. A ?2 agonist
4. A nonsteroidal an

3. A ?2 agonist
Rationale:
In treating an acute asthma attack, a short acting ?2 agonist such as albuterol (Proventil HFA) will be given to produce bronchodilation. Options 1, 2, and 4 are long-term control (preventive) medications.

192.) A nurse is collecting medication information from a client, and the client states that she is taking garlic as an herbal supplement. The nurse understands that the client is most likely treating which of the following conditions?
1. Eczema
2. Insomn

4. Hyperlipidemia
Rationale:
Garlic is an herbal supplement that is used to treat hyperlipidemia and hypertension. An herbal supplement that may be used to treat eczema is evening primrose. Insomnia has been treated with both valerian root and chamomile.

193.) Sodium hypochlorite (Dakin's solution) is prescribed for a client with a leg wound containing purulent drainage. The nurse is assisting in developing a plan of care for the client and includes which of the following in the plan?
1. Ensure that the s

1. Ensure that the solution is freshly prepared before use.
Rationale:
Dakin solution is a chloride solution that is used for irrigating and cleaning necrotic or purulent wounds. It can be used for packing necrotic wounds. It cannot be used to pack purule

194.) A nurse provides instructions to a client regarding the use of tretinoin (Retin-A). Which statement by the client indicates the need for further instructions?
1. "Optimal results will be seen after 6 weeks."
2. "I should apply a very thin layer to m

2. "I should apply a very thin layer to my skin."
Rationale:
Tretinoin is applied liberally to the skin. The hands are washed thoroughly immediately after applying. Therapeutic results should be seen after 2 to 3 weeks but may not be optimal until after 6

195.) A nurse is caring for a client who is taking metoprolol (Lopressor). The nurse measures the client's blood pressure (BP) and apical pulse (AP) immediately before administration. The client's BP is 122/78 mm/Hg and the AP is 58 beats/min. Based on th

1. Withhold the medication.
Rationale:
Metoprolol (Lopressor) is classified as a beta-adrenergic blocker and is used in the treatment of hypertension, angina, and myocardial infarction. Baseline nursing assessments include measurement of BP and AP immedia

196.) A client has been prescribed amikacin (Amikin). Which of the following priority baseline functions should be monitored?
1. Apical pulse
2. Liver function
3. Blood pressure
4. Hearing acuity

4. Hearing acuity
Rationale:
Amikacin (Amikin) is an antibiotic. This medication can cause ototoxicity and nephrotoxicity; therefore, hearing acuity tests and kidney function studies should be performed before the initiation of therapy. Apical pulse, live

197.) Collagenase (Santyl) is prescribed for a client with a severe burn to the hand. The nurse provides instructions to the client regarding the use of the medication. Which statement by the client indicates an accurate understanding of the use of this m

3. "I will apply the ointment once a day and cover it with a sterile dressing."
Rationale:
Collagenase is used to promote debridement of dermal lesions and severe burns. It is usually applied once daily and covered with a sterile dressing.

198.) Coal tar has been prescribed for a client with a diagnosis of psoriasis, and the nurse provides instructions to the client about the medication. Which statement by the client indicates a need for further instructions?
1. "The medication can cause ph

4. "The medication can cause systemic effects."
Rationale:
Coal tar is used to treat psoriasis and other chronic disorders of the skin. It suppresses DNA synthesis, mitotic activity, and cell proliferation. It has an unpleasant odor, can frequently stain

199.) A nurse is applying a topical glucocorticoid to a client with eczema. The nurse monitors for systemic absorption of the medication if the medication is being applied to which of the following body areas?
1. Back
2. Axilla
3. Soles of the feet
4. Pal

2. Axilla
Rationale:
Topical glucocorticoids can be absorbed into the systemic circulation. Absorption is higher from regions where the skin is especially permeable (scalp, axillae, face, eyelids, neck, perineum, genitalia), and lower from regions where p

200.) A client is seen in the clinic for complaints of skin itchiness that has been persistent over the past several weeks. Following data collection, it has been determined that the client has scabies. Lindane is prescribed, and the nurse is asked to pro

4. Leave the cream on for 8 to 12 hours and then remove by washing.
Rationale:
Lindane is applied in a thin layer to the entire body below the head. No more than 30 g (1 oz) should be used. The medication is removed by washing 8 to 12 hours later. Usually

201.) A nurse is preparing to administer eardrops to an infant. The nurse plans to:
1. Pull up and back on the ear and direct the solution onto the eardrum.
2. Pull down and back on the ear and direct the solution onto the eardrum.
3. Pull down and back o

3. Pull down and back on the ear and direct the solution toward the wall of the canal.
Rationale:
When administering eardrops to an infant, the nurse pulls the ear down and straight back. In the adult or a child older than 3 years, the ear is pulled up an

202.) A nurse is collecting data from a client about medications being taken, and the client tells the nurse that he is taking herbal supplements for the treatment of varicose veins. The nurse understands that the client is most likely taking which of the

1. Bilberry
Rationale:
Bilberry is an herbal supplement that has been used to treat varicose veins. This supplement has also been used to treat cataracts, retinopathy, diabetes mellitus, and peripheral vascular disease. Ginseng has been used to improve me

203.) A nurse is preparing to give the postcraniotomy client medication for incisional pain. The family asks the nurse why the client is receiving codeine sulfate and not "something stronger." In formulating a response, the nurse incorporates the understa

4. Does not alter respirations or mask neurological signs as do other opioids
Rationale:
Codeine sulfate is the opioid analgesic often used for clients after craniotomy. It is frequently combined with a nonopioid analgesic such as acetaminophen for added

204.) A client receives a dose of edrophonium (Enlon). The client shows improvement in muscle strength for a period of time following the injection. The nurse interprets that this finding is compatible with:
1. Multiple sclerosis
2. Myasthenia gravis
3. M

2. Myasthenia gravis
Rationale:
Myasthenia gravis can often be diagnosed based on clinical signs and symptoms. The diagnosis can be confirmed by injecting the client with a dose of edrophonium . This medication inhibits the breakdown of an enzyme in the n

205.) A nurse is assisting in preparing to administer acetylcysteine (Mucomyst) to a client with an overdose of acetaminophen (Tylenol). The nurse prepares to administer the medication by:
1. Administering the medication subcutaneously in the deltoid musc

4. Mixing the medication in a flavored ice drink and allowing the client to drink the medication through a straw
Rationale:
Because acetylcysteine has a pervasive odor of rotten eggs, it must be disguised in a flavored ice drink. It is consumed preferably

206.) A client is receiving baclofen (Lioresal) for muscle spasms caused by a spinal cord injury. The nurse monitors the client, knowing that which of the following is a side effect of this medication?
1. Muscle pain
2. Hypertension
3. Slurred speech
4. P

Rationale:
Side effects of baclofen include drowsiness, dizziness, weakness, and nausea. Occasional side effects include headache, paresthesia of the hands and feet, constipation or diarrhea, anorexia, hypotension, confusion, and nasal congestion. Paradox

207.) A client is suspected of having myasthenia gravis, and the health care provider administers edrophonium (Enlon) to determine the diagnosis. After administration of this medication, which of the following would indicate the presence of myasthenia gra

3. An increase in muscle strength
Rationale:
Edrophonium is a short-acting acetylcholinesterase inhibitor used as a diagnostic agent. When a client with suspected myasthenia gravis is given the medication intravenously, an increase in muscle strength woul

208.) A client with myasthenia gravis verbalizes complaints of feeling much weaker than normal. The health care provider plans to implement a diagnostic test to determine if the client is experiencing a myasthenic crisis and administers edrophonium (Enlon

auto-define "A client with myasthen..."
Rationale:
Edrophonium (Enlon) is administered to determine whether the client is reacting to an overdose of a medication (cholinergic crisis) or to an increasing severity of the disease (myasthenic crisis). When th

209.) A client with multiple sclerosis is receiving diazepam (Valium), a centrally acting skeletal muscle relaxant. Which of the following would indicate that the client is experiencing a side effect related to this medication?
1. Headache
2. Drowsiness
3

2. Drowsiness
Rationale:
Incoordination and drowsiness are common side effects resulting from this medication. Options 1, 3, and 4 are incorrect.

210.) Dantrolene (Dantrium) is prescribed for a client with a spinal cord injury for discomfort resulting from spasticity. The nurse tells the client about the importance of follow-up and the need for which blood study?
1. Creatinine level
2. Sedimentatio

3. Liver function studies
Rationale:
Dantrolene can cause liver damage, and the nurse should monitor liver function studies. Baseline liver function studies are done before therapy starts, and regular liver function studies are performed throughout therap

211.) A client with epilepsy is taking the prescribed dose of phenytoin (Dilantin) to control seizures. A phenytoin blood level is drawn, and the results reveal a level of 35 mcg/ml. Which of the following symptoms would be expected as a result of this la

3. Slurred speech
Rationale:
The therapeutic phenytoin level is 10 to 20 mcg/mL. At a level higher than 20 mcg/mL, involuntary movements of the eyeballs (nystagmus) appear. At a level higher than 30 mcg/mL, ataxia and slurred speech occur.

212.) Mannitol (Osmitrol) is being administered to a client with increased intracranial pressure following a head injury. The nurse assisting in caring for the client knows that which of the following indicates the therapeutic action of this medication?
1

4. Induces diuresis by raising the osmotic pressure of glomerular filtrate, thereby inhibiting tubular reabsorption of water and solutes
Rationale:
Mannitol is an osmotic diuretic that induces diuresis by raising the osmotic pressure of glomerular filtrat

213.) A client is admitted to the hospital with complaints of back spasms. The client states, "I have been taking two or three aspirin every 4 hours for the past week and it hasn't helped my back." Aspirin intoxication is suspected. Which of the following

1. Tinnitus
Rationale:
Mild intoxication with acetylsalicylic acid (aspirin) is called salicylism and is commonly experienced when the daily dosage is higher than 4 g. Tinnitus (ringing in the ears) is the most frequently occurring effect noted with intox

214.) A health care provider initiates carbidopa/levodopa (Sinemet) therapy for the client with Parkinson's disease. A few days after the client starts the medication, the client complains of nausea and vomiting. The nurse tells the client that:
1. Taking

2. Taking the medication with food will help to prevent the nausea.
Rationale:
If carbidopa/levodopa is causing nausea and vomiting, the nurse would tell the client that taking the medication with food will prevent the nausea. Additionally, the client sho

215.) A client with rheumatoid arthritis is taking acetylsalicylic acid (aspirin) on a daily basis. Which medication dose should the nurse expect the client to be taking?
1. 1 g daily
2. 4 g daily
3. 325 mg daily
4. 1000 mg daily

2. 4 g daily
Rationale:
Aspirin may be used to treat the client with rheumatoid arthritis. It may also be used to reduce the risk of recurrent transient ischemic attack (TIA) or brain attack (stroke) or reduce the risk of myocardial infarction (MI) in cli

216.) A nurse is caring for a client with gout who is taking Colcrys (colchicine). The client has been instructed to restrict the diet to low-purine foods. Which of the following foods should the nurse instruct the client to avoid while taking this medica

2. Scallops
Rationale:
Colchicine is a medication used for clients with gout to inhibit the reabsorption of uric acid by the kidney and promote excretion of uric acid in the urine. Uric acid is produced when purine is catabolized. Clients are instructed t

217.) A health care provider prescribes auranofin (Ridaura) for a client with rheumatoid arthritis. Which of the following would indicate to the nurse that the client is experiencing toxicity related to the medication?
1. Joint pain
2. Constipation
3. Rin

4. Complaints of a metallic taste in the mouth
Rationale:
Ridaura is the one gold preparation that is given orally rather than by injection. Gastrointestinal reactions including diarrhea, abdominal pain, nausea, and loss of appetite are common early in th

218.) A film-coated form of diflunisal has been prescribed for a client for the treatment of chronic rheumatoid arthritis. The client calls the clinic nurse because of difficulty swallowing the tablets. Which initial instruction should the nurse provide t

4. "Swallow the tablets with large amounts of water or milk."
Rationale:
Diflunisal may be given with water, milk, or meals. The tablets should not be crushed or broken open. Taking the medication with a large amount of water or milk should be tried befor

219.) A health care provider instructs a client with rheumatoid arthritis to take ibuprofen (Motrin). The nurse reinforces the instructions, knowing that the normal adult dose for this client is which of the following?
1. 100 mg orally twice a day
2. 200

3. 400 mg orally three times a day
Rationale:
For acute or chronic rheumatoid arthritis or osteoarthritis, the normal oral adult dose is 400 to 800 mg three or four times daily.

220.) A adult client with muscle spasms is taking an oral maintenance dose of baclofen (Lioresal). The nurse reviews the medication record, expecting that which dose should be prescribed?
1. 15 mg four times a day
2. 25 mg four times a day
3. 30 mg four t

1. 15 mg four times a day
Rationale:
Baclofen is dispensed in 10- and 20-mg tablets for oral use. Dosages are low initially and then gradually increased. Maintenance doses range from 15 to 20 mg administered three or four times a day.

221.) A nurse is reviewing the health care provider's prescriptions for an adult client who has been admitted to the hospital following a back injury. Carisoprodol (Soma) is prescribed for the client to relieve the muscle spasms; the health care provider

1. The normal adult dosage
Rationale:
The normal adult dosage for carisoprodol is 350 mg orally three or four times daily.

222.) A nurse has administered a dose of diazepam (Valium) to a client. The nurse would take which important action before leaving the client's room?
1. Giving the client a bedpan
2. Drawing the shades or blinds closed
3. Turning down the volume on the te

4. Per agency policy, putting up the side rails on the bed
Rationale:
Diazepam is a sedative-hypnotic with anticonvulsant and skeletal muscle relaxant properties. The nurse should institute safety measures before leaving the client's room to ensure that t

223.) A client with a psychotic disorder is being treated with haloperidol (Haldol). Which of the following would indicate the presence of a toxic effect of this medication?
1. Nausea
2. Hypotension
3. Blurred vision
4. Excessive salivation

4. Excessive salivation
Rationale:
Toxic effects include extrapyramidal symptoms (EPS) noted as marked drowsiness and lethargy, excessive salivation, and a fixed stare. Akathisia, acute dystonias, and tardive dyskinesia are also signs of toxicity. Hypoten

224.) Neuroleptic malignant syndrome is suspected in a client who is taking chlorpromazine. Which medication would the nurse prepare in anticipation of being prescribed to treat this adverse effect related to the use of chlorpromazine?
1. Protamine sulfat

2. Bromocriptine (Parlodel)
Rationale:
Bromocriptine is an antiparkinsonian prolactin inhibitor used in the treatment of neuroleptic malignant syndrome. Vitamin K is the antidote for warfarin (Coumadin) overdose. Protamine sulfate is the antidote for hepa

225.) A nursing student is assigned to care for a client with a diagnosis of schizophrenia. Haloperidol (Haldol) is prescribed for the client, and the nursing instructor asks the student to describe the action of the medication. Which statement by the nur

4. It blocks the binding of dopamine to the postsynaptic dopamine receptors in the brain.
Rationale:
Haloperidol acts by blocking the binding of dopamine to the postsynaptic dopamine receptors in the brain. Imipramine hydrochloride (Tofranil) blocks the r

226.) A client receiving lithium carbonate (Lithobid) complains of loose, watery stools and difficulty walking. The nurse would expect the serum lithium level to be which of the following?
1. 0.7 mEq/L
2. 1.0 mEq/L
3. 1.2 mEq/L
4. 1.7 mEq/L

4. 1.7 mEq/L
Rationale:
The therapeutic serum level of lithium ranges from 0.6 to 1.2 mEq/L. Serum lithium levels above the therapeutic level will produce signs of toxicity.

227.) When teaching a client who is being started on imipramine hydrochloride (Tofranil), the nurse would inform the client that the desired effects of the medication may:
1. Start during the first week of administration
2. Not occur for 2 to 3 weeks of a

2. Not occur for 2 to 3 weeks of administration
Rationale:
The therapeutic effects of administration of imipramine hydrochloride may not occur for 2 to 3 weeks after the antidepressant therapy has been initiated. Therefore options 1, 3, and 4 are incorrec

228.) A client receiving an anxiolytic medication complains that he feels very "faint" when he tries to get out of bed in the morning. The nurse recognizes this complaint as a symptom of:
1. Cardiac dysrhythmias
2. Postural hypotension
3. Psychosomatic sy

2. Postural hypotension
Rationale:
Anxiolytic medications can cause postural hypotension. The client needs to be taught to rise to a sitting position and get out of bed slowly because of this adverse effect related to the medication. Options 1, 3, and 4 a

229.) A client who is taking lithium carbonate (Lithobid) is scheduled for surgery. The nurse informs the client that:
1. The medication will be discontinued a week before the surgery and resumed 1 week postoperatively.
2. The medication is to be taken un

3. The medication will be discontinued 1 to 2 days before the surgery and resumed as soon as full oral intake is allowed.
Rationale:
The client who is on lithium carbonate must be off the medication for 1 to 2 days before a scheduled surgical procedure an

230.) A client is placed on chloral hydrate (Somnote) for short-term treatment. Which nursing action indicates an understanding of the major side effect of this medication?
1. Monitoring neurological signs every 2 hours
2. Monitoring the blood pressure ev

3. Instructing the client to call for ambulation assistance
Rationale:
Chloral hydrate (a sedative-hypnotic) causes sedation and impairment of motor coordination; therefore, safety measures need to be implemented. The client is instructed to call for assi

231.) A client admitted to the hospital gives the nurse a bottle of clomipramine (Anafranil). The nurse notes that the medication has not been taken by the client in 2 months. What behaviors observed in the client would validate noncompliance with this me

4. Frequent handwashing with hot, soapy water
Rationale:
Clomipramine is commonly used in the treatment of obsessive-compulsive disorder. Handwashing is a common obsessive-compulsive behavior. Weight gain is a common side effect of this medication. Tachyc

232.) A client in the mental health unit is administered haloperidol (Haldol). The nurse would check which of the following to determine medication effectiveness?
1. The client's vital signs
2. The client's nutritional intake
3. The physical safety of oth

4. The client's orientation and delusional status
Rationale:
Haloperidol is used to treat clients exhibiting psychotic features. Therefore, to determine medication effectiveness, the nurse would check the client's orientation and delusional status. Vital

233.) Diphenhydramine hydrochloride (Benadryl) is used in the treatment of allergic rhinitis for a hospitalized client with a chronic psychotic disorder. The client asks the nurse why the medication is being discontinued before hospital discharge. The nur

3. Addictive properties are enhanced in the presence of psychotropic medications.
Rationale:
The addictive properties of diphenhydramine hydrochloride are enhanced when used with psychotropic medications. Allergic symptoms may not be short term and will o

234.) A hospitalized client is started on phenelzine sulfate (Nardil) for the treatment of depression. At lunchtime, a tray is delivered to the client. Which food item on the tray will the nurse remove?
1. Yogurt
2. Crackers
3. Tossed salad
4. Oatmeal coo

1. Yogurt
Rationale:
Phenelzine sulfate is a monoamine oxidase inhibitor (MAOI). The client should avoid taking in foods that are high in tyramine. These foods could trigger a potentially fatal hypertensive crisis. Foods to avoid include yogurt, aged chee

235.) A tricyclic antidepressant is administered to a client daily. The nurse plans to monitor for the common side effects of the medication and includes which of the following in the plan of care?
1. Offer hard candy or gum periodically.
2. Offer a nutri

1. Offer hard candy or gum periodically.
Rationale:
Dry mouth is a common side effect of tricyclic antidepressants. Frequent mouth rinsing with water, sucking on hard candy, and chewing gum will alleviate this common side effect. It is not necessary to mo

236.) A client is being treated for depression with amitriptyline hydrochloride. During the initial phases of treatment, the most important nursing intervention is:
1. Prescribing the client a tyramine-free diet
2. Checking the client for anticholinergic

4. Getting baseline postural blood pressures before administering the medication and each time the medication is administered
Rationale:
Amitriptyline hydrochloride is a tricyclic antidepressant often used to treat depression. It causes orthostatic change

237.) A client who is on lithium carbonate (Lithobid) will be discharged at the end of the week. In formulating a discharge teaching plan, the nurse will instruct the client that it is most important to:
1. Avoid soy sauce, wine, and aged cheese.
2. Have

4. Check with the psychiatrist before using any over-the-counter (OTC) medications or prescription medications.
Rationale:
Lithium is the medication of choice to treat manic-depressive illness. Many OTC medications interact with lithium, and the client is

238.) Ribavirin (Virazole) is prescribed for the hospitalized child with respiratory syncytial virus (RSV). The nurse prepares to administer this medication via which of the following routes?
1. Orally
2. Via face mask
3. Intravenously
4. Intramuscularly

2. Via face mask
Rationale:
Ribavirin is an antiviral respiratory medication used mainly in hospitalized children with severe RSV and in high-risk children. Administration is via hood, face mask, or oxygen tent. The medication is most effective if adminis

239.) Which of the following precautions will the nurse specifically take during the administration of ribavirin (Virazole) to a child with respiratory syncytial virus (RSV)?
1. Wearing goggles
2. Wearing a gown
3. Wearing a gown and a mask
4. Handwashing

1. Wearing goggles
Rationale:
Some caregivers experience headaches, burning nasal passages and eyes, and crystallization of soft contact lenses as a result of administration of ribavirin. Specific to this medication is the use of goggles. A gown is not ne

240.) A client with Parkinson's disease has been prescribed benztropine (Cogentin). The nurse monitors for which gastrointestinal (GI) side effect of this medication?
1. Diarrhea
2. Dry mouth
3. Increased appetite
4. Hyperactive bowel sounds

2. Dry mouth
Rationale:
Common GI side effects of benztropine therapy include constipation and dry mouth. Other GI side effects include nausea and ileus. These effects are the result of the anticholinergic properties of the medication.
*
Eliminate options

241.) A client with a history of simple partial seizures is taking clorazepate (Tranxene), and asks the nurse if there is a risk of addiction. The nurse's response is based on the understanding that clorazepate:
1. Is not habit forming, either physically

3. Leads to physical and psychological dependence with prolonged high-dose therapy
Rationale:
Clorazepate is classified as an anticonvulsant, antianxiety agent, and sedative-hypnotic (benzodiazepine). One of the concerns with clorazepate therapy is that t

242.) A client who was started on anticonvulsant therapy with clonazepam (Klonopin) tells the nurse of increasing clumsiness and unsteadiness since starting the medication. The client is visibly upset by these manifestations and asks the nurse what to do.

4. Are worse during initial therapy and decrease or disappear with long-term use
Rationale:
Drowsiness, unsteadiness, and clumsiness are expected effects of the medication during early therapy. They are dose related and usually diminish or disappear altog

243.) A hospitalized client is having the dosage of clonazepam (Klonopin) adjusted. The nurse should plan to:
1. Weigh the client daily.
2. Observe for ecchymosis.
3. Institute seizure precautions.
4. Monitor blood glucose levels.

3. Institute seizure precautions.
Rationale:
Clonazepam is a benzodiazepine used as an anticonvulsant. During initial therapy and during periods of dosage adjustment, the nurse should initiate seizure precautions for the client. Options 1, 2, and 4 are no

244.) A client has a prescription for valproic acid (Depakene) orally once daily. The nurse plans to:
1. Administer the medication with an antacid.
2. Administer the medication with a carbonated beverage.
3. Ensure that the medication is administered at t

3. Ensure that the medication is administered at the same time each day.
Rationale:
Valproic acid is an anticonvulsant, antimanic, and antimigraine medication. It may be administered with or without food. It should not be taken with an antacid or carbonat

245.) A client taking carbamazepine (Tegretol) asks the nurse what to do if he misses one dose. The nurse responds that the carbamazepine should be:
1. Withheld until the next scheduled dose
2. Withheld and the health care provider is notified immediately

3. Taken as long as it is not immediately before the next dose
Rationale:
Carbamazepine is an anticonvulsant that should be taken around the clock, precisely as directed. If a dose is omitted, the client should take the dose as soon as it is remembered, a

b. Avoid driving a motor vehicle until stabilized on the drug.

A client tells the nurse that he has started to take an OTC antihistamine, diphenhydramine. In teaching him about side effects, what is most important for the nurse to tell the client?
a. Do not to take this drug at bedtime to avoid insomnia.
b. Avoid dri

c. Acute pharyngitis.

The client complains of a sore throat and has been told it is due to beta-hemolytic streptococcal infection. The nurse realizes this condition is called what?
a. Acute rhinitis.
b. Acute sinusitis.
c. Acute pharyngitis.
d. Acute rhinorrhea.

d. Limit the drug to 5 days of use to prevent rebound nasal congestion.

A client is prescribed the decongestant oxymetazoline (Afrin) nasal spray. What should the nurse teach the client?
a. Take this drug at bedtime as a sleep aid.
b. Directly spray away from the nasal septum and gently sniff.
c. This drug may be used in main

b. To loosen bronchial secretions so they can be eliminated by coughing

A client has been prescribed guaifenesin (Robitussin). The nurse realizes that the purpose of the drug is to accomplish what?
a. To treat allergic rhinitis and prevent motion sickness
b. To loosen bronchial secretions so they can be eliminated by coughing

d. Dry nasal mucosa

Beclomethasone (Beconase) has been prescribed for a client with allergic rhinitis. The nurse teaches the client that which is the most common side effect from continuous use?
a. Dizziness
b. Rhinorrhea
c. Hallucinations
d. Dry nasal mucosa

a. Take medication with food to decrease gastric distress.
b. Avoid alcohol and other central nervous system depressants.
c. Notify the health care provider if confusion or hypotension occurs.
d. Take sugarless candy, gum, or ice chips for temporary relie

The nurse is teaching a client about diphenhydramine (Benadryl). Which are topics to include? (Select all that apply.)
a. Take medication with food to decrease gastric distress.
b. Avoid alcohol and other central nervous system depressants.
c. Notify the

a. "Do not drive after taking this medication.

The nurse is caring for a client who is taking a first-generation antihistamine. What is the most important fact for the nurse to teach the client?
a. "Do not drive after taking this medication."
b. "Make sure you drink a lot of liquids while on this medi

d. "You may be able to safely take a second-generation antihistamine.

The nurse is caring for a client in the clinic who states that he is afraid of taking antihistamines because he is a truck driver. What is the best information for the nurse to give this client?
a. "Take the medication only when you are not driving."
b.

Administer guaifenesin.

The client tells the nurse that she has a bad cold, is coughing, and feels like she has "stuff" in her lungs. What should the nurse do?
a. Administer dextromethorphan.
b. Administer guaifenesin.
c. Encourage the client to drink fluids hourly.
d. Administe

b. This medication has fewer sedative effects.

What is the most important thing for the nurse to teach a client who is switching allergy medications from diphenhydramine (Benadryl) to loratadine (Claritin)?
a. This medication can potentially cause dysrhythmias.
b. This medication has fewer sedative ef

b. "Overuse of nasal decongestants results in rebound congestion.

A client complains of worsening nasal congestion despite the use of oxymetazoline (Afrin) nasal spray every 2 hours. What is the nurse's best response?
a. "Oxymetazoline is not an effective nasal decongestant."
b. "Overuse of nasal decongestants results i

b. "This medication will help prevent the inflammatory response of my allergies.

Which statement indicates that the client understands the teaching about beclomethasone diproprionate (Beconase)?
a. "I will need to taper off the medication to prevent acute adrenal crisis."
b. "This medication will help prevent the inflammatory response

a. "This medication may cause drowsiness and dizziness.

A client is prescribed an antitussive medication. What is the most important thing for the nurse to teach the client?
a. "This medication may cause drowsiness and dizziness."
b. "Watch out for diarrhea and abdominal cramping."
c. "This may cause tremors a

c. Increase fluid intake in order to decrease viscosity of secretions.

Which is the best instruction for the nurse to include when teaching a client about the use of expectorants?
a. Restrict fluids in order to decrease mucus production.
b. Take the medication once a day only, at bedtime.
c. Increase fluid intake in order to

a. Asthma

A client is diagnosed with a pulmonary disorder that causes COPD. Lungs tissue changes are normally reversible with this condition. The nurse understands that which is the client's most likely diagnosis?
a. Asthma
b. Emphysema
c. Bronchiectasis
d. Chronic

b. epinephrine (Adrenalin)

A client with COPD has an acute bronchospasm. The nurse knows that which is the best medication for this emergency situation?
a. zafirlukast (Accolate)
b. epinephrine (Adrenalin)
c. dexamethasone (Decadron)
d. oxtriphylline-theophyllinate (Choledyl)

c. Increased heart rate

A client is taking aminophylline-theophylline ethylenediamine (Somophyllin). For what should the nurse monitor the client?
a. Drowsiness
b. Hypoglycemia
c. Increased heart rate
d. Decreased white blood cell count

b. 10 to 20 mcg/mL

A client is prescribed theophylline to relax the smooth muscles of the bronchi. The nurse monitors the client's theophylline serum levels to maintain which therapeutic range?
a. 1 to 10 mcg/mL
b. 10 to 20 mcg/mL
c. 20 to 30 mcg/mL
d. 30 to 40 mcg/mL

a. Maintenance treatment of asthma

A client with COPD is taking a leukotriene antagonist, montelukast (Singulair). The nurse is aware that this medication is given for which purpose?
a. Maintenance treatment of asthma
b. Treatment of an acute asthma attack
c. Reversing bronchospasm associa

c. Continue to assess the client's oxygenation.

The nurse is caring for a client with a theophylline level of 14 mcg/mL. What is the priority nursing intervention?
a. Increase the IV drip rate.
b. Monitor the client for toxicity.
c. Continue to assess the client's oxygenation.
d. Stop the IV for an hou

d. Tachycardia

Discharge teaching to a client receiving a beta-agonist bronchodilator should emphasize reporting which side effect?
a. Hypoglycemia
b. Nonproductive cough
c. Sedation
d. Tachycardia

d. St. John's wort

The nurse instructs the client to avoid which over-the-counter products when taking theophylline (Theo-Dur)?
a. acetaminophen (Tylenol)
b. echinacea
c. diphenhydramine (Benadryl)
d. St. John's wort

a. Monitor client for potential chest pain.

A nurse reviews a client's medication history and notes that the client is taking a nonselective adrenergic agonist bronchodilator and has a history of coronary artery disease. What is a priority nursing intervention?
a. Monitor client for potential chest

d. Salmeterol has a longer duration of action.

The nurse is instructing a client about the advantages of salmeterol (Serevent) over other beta2 agonists such as albuterol (Proventil). How will the nurse explain to the client the difference in these two medications?
a. Salmeterol has a shorter onset of

c. "This medication will prevent the inflammation that causes your asthma attack.

Client teaching regarding the use of antileukotriene agents such as zafirlukast (Accolate) should include which statement?
a. "Take the medication as soon as you begin wheezing."
b. "It will take about 3 weeks before you notice a therapeutic effect."
c.

c. Administer a beta2 adrenergic agonist.

A client with a history of asthma is short of breath and says, "I feel like I'm having an asthmatic attack." What is the nurse's best action?
a. Call a code.
b. Ask the client to describe the symptoms.
c. Administer a beta2 adrenergic agonist.
d. Administ

a. Monitor for heart rate >100 beats/min.

A client has taken metaproterenol. What is the nurse's priority action?
a. Monitor for heart rate >100 beats/min.
b. Tell the client not to drive for 2 hours.
c. Monitor for sedation.
d. Assess for elevated blood pressure.

b. Rinse his mouth with water after each use.

A client demonstrates understanding of flunisolide (AeroBid) by saying that he will do what?
a. Take two puffs to treat an acute asthma attack.
b. Rinse his mouth with water after each use.
c. Immediately stop taking his oral prednisone when he starts usi

d. Teach the child to use a spacer.

The nurse is caring for a young child who has been prescribed an inhaler for control of her asthma. The child is having difficulty using the inhaler. What is the nurse's best action?
a. Tell the parent to hold the inhaler for the child.
b. Ask the health

d. The client with atrial fibrillation with a rate of 100

The nurse is caring for clients on the pulmonary unit. Which client should not receive epinephrine if ordered?
a. The client with a history of emphysema
b. The client with a history of type 2 diabetes
c. The client who is 16 years old
d. The client with a

b. Administer the albuterol first, wait 5 minutes, and administer ipratropium bromide, followed by beclomethasone several minutes later.

The health care provider orders ipratropium bromide (Atrovent), albuterol (Proventil), and beclomethasone (Vanceril) inhalers for a client. What is the nurse's best action?
a. Question the order; three inhalers should not be given at one time.
b. Administ

c. "Hold your breath for 10 seconds if you can after you inhale the medication.

Which instruction will the nurse include when teaching a client about the proper use of metered-dose inhalers?
a. "After you inhale the medication once, repeat until you obtain relief."
b. "Make sure that you puff out air repeatedly after you inhale the m

c. Liquefying and loosening of bronchial secretions

What will the nurse expect to find that would indicate a therapeutic effect of acetylcysteine (Mucomyst)?
a. Decreased cough reflex
b. Decreased nasal secretions
c. Liquefying and loosening of bronchial secretions
d. Relief of bronchospasms

c. Monitor blood glucose levels every 4 hours when taking albuterol.

What is the most important thing for the nurse to teach the client with a history of diabetes and asthma who has started on albuterol PRN?
a. Take Tylenol for headaches when taking albuterol.
b. Monitor for orthostatic hypotension every 2 hours when takin

b. "Take the ipratropium at least 5 minutes before the cromolyn.

A client is prescribed ipratropium and cromolyn sodium. What will the nurse teach the client?
a. "Do not take these medications within 4 hours of each other."
b. "Take the ipratropium at least 5 minutes before the cromolyn."
c. "Administer both medication

c. Hold the next dose of theophylline.

A client taking an oral theophylline preparation is due for her next dose and has a blood pressure of 100/50 mm Hg and a heart rate of 110. The client is irritable. What is the best action for the nurse to take?
a. Continue to monitor the client.
b. Call

b. Lack of exercise

A client complains of constipation and requires a laxative. In providing teaching to the client, the nurse reviews the common causes of constipation, including which cause?
a. Motion sickness
b. Lack of exercise
c. Food intolerance
d. Bacteria (Escherichi

b. Block serotonin receptors in the CTZ

A client has nausea and is taking ondansetron (Zofran). The nurse explains that the action of this drug is what?
a. Stimulate the CTZ
b. Block serotonin receptors in the CTZ
c. Block dopamine receptors in the CTZ
d. Coat the wall of the GI tract and absor

a. Acts on smooth intestinal muscle to gently increase peristalsis

A client who has constipation is prescribed a bisacodyl suppository. The nurse explains that bisacodyl does what?
a. Acts on smooth intestinal muscle to gently increase peristalsis
b. Absorbs water into the intestines to increase bulk and peristalsis
c. L

d. Dry mouth

A client is using the scopolamine patch to prevent motion sickness. The nurse teaches the client that which is a common side effect of this drug?
a. Diarrhea
b. Vomiting
c. Insomnia
d. Dry mouth

c. Alcohol

When metoclopramide (Raglan) is given for nausea, the client is cautioned to avoid which substance?
a. Milk
b. MAOIs
c. Alcohol
d. Carbonated beverages

a. Warn the client to avoid laxative abuse.
b. Record the frequency of bowel movements.
c. Warn the client against taking sedatives concurrently.
d. Encourage the client to increase fluids.
e. Instruct the client to avoid this drug if he or she has narrow

The nurse is administering opium tincture (paregoric) to a client. Which should be included in the client teaching regarding this medication? (Select all that apply.)
a. Warn the client to avoid laxative abuse.
b. Record the frequency of bowel movements.

a. Client has not had a bowel movement in 3 days.

Which assessment finding will need intervention and is related to the client's use of aluminum hydroxide (Amphojel)?
a. Client has not had a bowel movement in 3 days.
b. Client has had one loose stool this week.
c. Client is complaining of gastric upset.

c. Client taking magnesium-containing antacids who has renal failure.

Which client needs immediate intervention?
a. Client taking aluminum-containing antacids with complaints of reflux.
b. Client taking calcium-containing antacids who is hypocalcemic.
c. Client taking magnesium-containing antacids who has renal failure.
d.

a. Assess for metabolic alkalosis.

What assessment has the highest priority for a client using sodium bicarbonate to treat gastric hyperacidity?
a. Assess for metabolic alkalosis.
b. Assess for fluid volume deficit.
c. Assess for hyperkalemia.
d. Assess for hypercalcemia.

b. Potential risk for bleeding related to thrombocytopenia

Which nursing diagnoses is appropriate for a client receiving famotidine (Pepcid)?
a. Increased risk for infection related to immunosuppression
b. Potential risk for bleeding related to thrombocytopenia
c. Alteration in urinary elimination related to rete

b. "Smoking decreases the effects of this medication, so I should look into cessation programs.

Which statement demonstrates to the nurse that the client understands instructions regarding the use of histamine2-receptor antagonists?
a. "Since I am taking this medication, it is all right for me to eat spicy foods."
b. "Smoking decreases the effects o

d. Administer the medications and assess the client for relief.

A client is prescribed Lorazepam (Ativan) and a glucocorticoid during chemotherapy treatments. What is the nurse's best action?
a. Call the health care provider and question the order.
b. Only administer the Ativan if the client seems anxious.
c. Administ

d. pantoprazole (Protonix)

A nurse is caring for a client who is unable to tolerate oral medications. The nurse anticipates that the client may be prescribed which proton pump inhibitor to be administered intravenously?
a. esomeprazole (Nexium)
b. lansoprazole (Prevacid)
c. omepraz

c. "I will apply the scopolamine patches to rotating sites on my arms.

Which client statement indicates that further teaching is needed?
a. "I will not drive while I am taking these medications because they may cause drowsiness."
b. "I may take Tylenol to treat the headache caused by ondansetron (Zofran)."
c. "I will apply t

b. Gastric assessment

The nurse is administering loperamide (Imodium) to a client with diarrhea. What assessment is essential for this client?
a. Vascular assessment
b. Gastric assessment
c. Hourly blood pressure measurements
d. White blood count

c. Decrease in gastric motility

Which outcome assessment is essential to monitor for the client taking diphenoxylate (Lomotil)?
a. Increase in bowel sounds
b. Increase in number of bowel movements
c. Decrease in gastric motility
d. Decrease in urination

c. Administer 30 minutes before meals and at bedtime.

The nurse is planning to administer metoclopramide (Reglan). What is a primary intervention?
a. Administer with food to decrease gastrointestinal upset.
b. Administer every 6 hours around the clock.
c. Administer 30 minutes before meals and at bedtime.
d.

c. Combination therapy blocks different vomiting pathways.

What will the nurse teach the client about the reason for administering multiple medications for relief of nausea and vomiting?
a. Combination therapy decreases the risk of constipation.
b. Combination therapy is more cost-effective.
c. Combination therap

b. Fluid volume deficit related to nausea and vomiting

In developing a plan of care for a client receiving an antihistamine antiemetic agent, which nursing diagnosis would be of highest priority?
a. Knowledge deficit regarding medication administration
b. Fluid volume deficit related to nausea and vomiting
c.

c. "Brush your teeth and gargle to help with dryness in your mouth.

What instruction is most important for the nurse to teach a client who is taking an anticholinergic agent to treat nausea and vomiting?
a. "Assess your stools for dark streaks."
b. "Do not take more than two doses of this medication."
c. "Brush your teeth

b. Weigh the client before chemotherapy.

A client is prescribed granisetron (Kytril) IV for relief of nausea and vomiting caused by cancer chemotherapy. What intervention is most appropriate for this client?
a. Administer the medication at least 12 hours before the start of chemotherapy.
b. Weig

b. Administer ondansetron HCL (Zofran) 30 minutes before therapy and two doses after therapy.

A client is starting cisplatin therapy for cancer. What intervention is appropriate for this client?
a. Administer granisetron (Kytril) 60 minutes before therapy and for several days after surgery.
b. Administer ondansetron HCL (Zofran) 30 minutes before

c. Evaluate renal function.

Before administering a stimulant laxative to a client, which nursing intervention is the priority?
a. Obtain a history of constipation and causes.
b. Record baseline vital signs.
c. Evaluate renal function.
d. Assess fluid and electrolyte balance.

c. Monitor signs and symptoms of fluid and electrolyte imbalance.

Which assessment is most important for the client who is taking stimulant laxatives?
a. Monitor bowel elimination daily.
b. Monitor intake and output.
c. Monitor signs and symptoms of fluid and electrolyte imbalance.
d. Monitor heart rate and blood pressu

d. "After 3 days, switch patch to alternate ear."
e. "Apply patch 4 hours before effect is desired."
f. "Drowsiness is a concern while on this medication.

A client is prescribed scopolamine. What information will the nurse include on the teaching plan for this client? (Select all that apply.)
a. "Do not take this medication if you are dizzy."
b. "Do not use laxatives while on this medication."
c. "Do not us

a. Helicobacter pylori

A client is diagnosed with peptic ulcer disease. The nurse realizes that which factor is a predisposing factor for this condition?
a. Helicobacter pylori
b. hyposecretion of pepsin
c. decreased hydrochloric acid
d. decreased number of parietal cells

d. To combine with protein to form a viscous substance that forms a protective covering of ulcer

When a client is given sucralfate (Carafate), the nurse knows that its mode of action is what?
a. To neutralize gastric acidity
b. To inhibit gastric acid secretion by inhibiting histamine at H2 receptors in parietal cells
c. To suppress gastric acid secr

c. The drug must be administered separate from an antacid by at least 1 hour
e. Smoking should be avoided while taking this drug
f. Foods high in vitamin B12 should be increased in diet

A client is taking ranitidine (Zantac). The nurse who is teaching the client about this drug should include which information? (Select all that apply.)
a. Drug-induced impotence is irreversible
b. The drug must be administered 30 minutes before meals
c. T

d. Antacids neutralize HCl and reduce pepsin activity.

When a client complains of pain accompanying a peptic ulcer, why should an antacid be given?
a. Antacids decrease GI motility.
b. Antacids decrease gastric acid secretion.
c. Aluminum hydroxide is a systemic antacid.
d. Antacids neutralize HCl and reduce

b. Dizziness
d. Headaches
f. Decreased libido

A client is taking famotidine (Pepcid) to inhibit gastric secretions. What are the side effects of famotidine? (Select all that apply.)
a. Diarrhea
b. Dizziness
c. Dry mouth
d. Headaches
e. Blurred vision
f. Decreased libido

b. "I will drink 2 ounces of water after taking aluminum hydroxide.

A client has just been prescribed aluminum hydroxide (Amphojel, ALternaGEL, Alu-Tab) for peptic ulcer pain. The nurse has provided instructions to the client. Which statement by the client indicates to the nurse that the client understands the instruction

a. Administer just before meals.

What is a priority nursing intervention when administering ranitidine (Zantac)?
a. Administer just before meals.
b. Administer right after eating.
c. Administer 1 to 2 hours after meals.
d. Administer during meals.

c. The client has no throat pain.

The health care provider prescribes lansoprazole (Prevacid) to a client. Which assessment indicates to the nurse that the medication has had a therapeutic effect?
a. The client has no diarrhea.
b. The client has no gastric pain.
c. The client has no throa

b. Absent bowel sounds, hard abdomen

The nurse is caring for a client who is taking sucralfate (Carafate, Sulcrate) for treatment of a duodenal ulcer. Which assessment requires action by the nurse?
a. Sodium level 140 mEq/L
b. Absent bowel sounds, hard abdomen
c. Urinary output 30 mL/hr
d. C

c. Allow the tablet to dissolve in water before administering.

When administering sucralfate (Carafate) to a client with a nasogastric tube, what is an essential intervention?
a. Crush the tablet into a fine powder before mixing it with water.
b. Administer with a bolus tube feeding.
c. Allow the tablet to dissolve i

c. "This medication will form a protective barrier over the gastric mucosa.

What information should the nurse include in a teaching plan for the client who is prescribed sucralfate (Carafate)?
a. "This medication will neutralize gastric acid."
b. "This medication will enhance gastric absorption of meals."
c. "This medication will

b. Administer misoprostol.
d. Instruct the client to take omeprazole with the aspirin.

The nurse is caring for a client who is experiencing gastric distress from the long-term use of aspirin for treatment of arthritis. What is the best intervention for this client? (Select all that apply.)
a. Stop all aspirin therapy.
b. Administer misopros

a. Dehydration

The nurse reviews the client's list of medication, which includes mannitol. The nurse must be aware that which condition is a contraindication for use of this drug?
a. Dehydration
b. Kidney stones
c. Eczema
d. Gout

c. Cyclopentolate

The client is being prepared for an eye examination. When the nurse takes the health history, the client says that she is sensitive to atropine sulfate. What drug might be used instead for the examination?
a. Diclofenac
b. Suprofen
c. Cyclopentolate
d. Be

c. Electrolytes

An 85-year-old client is taking acetazolamide, a carbonic anhydrase inhibitor. A nursing intervention associated with clients receiving this drug is to monitor what?
a. Weight
b. Complete blood count
c. Electrolytes
d. Urine output

d. travoprost

The nurse reviews the African-American client's list of medications. It is important for the nurse to be aware that the prostaglandin analogue more effective in African Americans than in non-African Americans is wha?
a. latanoprost
b. bimatoprost
c. unopr

a. School-aged children may need only one drug, not a combination.

The school nurse is preparing a presentation for the parent-teacher association meeting on medications commonly used in school-aged children. It is important to note what primary disadvantage of the use of combination products such as Cortisporin Otic?
a.

a. Hydrogen peroxide

The camp nurse reviews the "shopping list" of supplies needed for the upcoming camping season. What product is recommended to prevent and treat chronic impaction of cerumen?
a. Hydrogen peroxide
b. Rubbing alcohol
c. Charcoal
d. Salt solution

a. Instruct the client to report changes in vision and breathing.
b. Maintain sterile technique and prevent dropper

The nurse prepares a health teaching plan for the client with glaucoma. Which important nursing intervention are included for this client? (Select all that apply.)
a. Instruct the client to report changes in vision and breathing.
b. Maintain sterile techn

d. Client's pupils are constricted to 2 mm.

The nurse administers pilocarpine (Pilocar) to a client with glaucoma. Which assessment finding would indicate a therapeutic effect of the medication?
a. Client's eyes appear clear, without drainage.
b. Client states that her eyes feel very dry.
c. Client

c. "I should rinse the eye dropper with tap water after each use.

Which statement, made by a client, indicates to the nurse a need for further client teaching regarding proper administration of eye drops?
a. "I will put pressure on the inside corner of my eye after I administer the drops."
b. "I will be careful not to t

b. Warm the eardrops to room temperature before administration.

The nurse is planning to administer eardrops. Which intervention is essential to include in the plan of care?
a. Eardrops should be cool when being administered.
b. Warm the eardrops to room temperature before administration.
c. The pinna of an adult shou

b. carbamide peroxide

A client is complaining of excessive earwax that diminishes hearing ability. What medication will the nurse use to assist the client?
a. acetic acid
b. carbamide peroxide
c. hydrocortisone
d. glycerin

b. Instruct the client that one drop is optimal.

The nurse evaluates the client using eyedrops. The client puts two drops into his eye. What is the nurse's best action?
a. Continue to observe the client.
b. Instruct the client that one drop is optimal.
c. Have the client irrigate his eye to remove exces

b. A horny layer of epidermis

The nurse reviews the client's list of medications and recalls that the purpose of keratolytic agents is to remove what?
a. A horny layer of dermis
b. A horny layer of epidermis
c. Erythematous lesions
d. Hair follicles

a. Avoid sunlight.
c. Monitor CBC, glucose, and lipids.
d. Do not breastfeed or give blood.

Nursing implications for health teaching with clients taking isotretinoin include which implications? (Select all that apply.)
a. Avoid sunlight.
b. Monitor weight
c. Monitor CBC, glucose, and lipids.
d. Do not breastfeed or give blood.

c. Assess lesions

The nurse is doing health teaching with a client with psoriasis. Which is a nursing implication of the new biologic agents for the management of psoriasis?
a. Daily weight
b. Monitor electrolytes
c. Assess lesions
d. Monitor CBC and T-cell count

d. finasteride

A 55-year-old man has a chief complaint: "I'm going bald." Which drug is used to treat male pattern baldness?
a. dexamethasone
b. PABA
c. minoxidil
d. finasteride

a. Metabolic acidosis
c. Respiratory alkalosis

The client has second- and third-degree burns over 25% of his body. Mafenide acetate has been ordered. What acid-base imbalance can result from its use? (Select all that apply.)
a. Metabolic acidosis
b. Metabolic alkalosis
c. Respiratory alkalosis
d. Resp

b. Thinning of the skin
d. Purpura

The nurse reviews the client's medication history. Based on the client's prolonged use of glucocorticoids, what does the assessment include? (Select all that apply.)
a. Obesity
b. Thinning of the skin
c. Erythematous lesions
d. Purpura

b. A review of iPLEDGE educational materials
c. That a negative pregnancy test is required before each monthly refill

A 20-year-old woman comes to the clinic for follow-up related to isotretinoin use. The nurse reviews the iPLEDGE program, which includes which important information? (Select all that apply.)
a. That an effective method of contraception must be used throug

a. Sunscreen products should contain information about UVA and UVB SPF protection.
b. UVB radiation is greatest between 10 AM and 4 PM.
d. SPF should be at least 15 in sunscreen products.

The school nurse prepares a program for junior high school students on sun safety. What is important information to include? (Select all that apply.)
a. Sunscreen products should contain information about UVA and UVB SPF protection.
b. UVB radiation is gr

d. Silver sulfadiazine cream

Which intervention is most appropriate for the client with second-degree burns?
a. IV antibiotics
b. Isolation
c. IV dextrose infusion
d. Silver sulfadiazine cream

a. Perform pregnancy test.

A 20-year-old client is starting isotretinoin (Accutane) therapy. What is an essential nursing intervention for this client?
a. Perform pregnancy test.
b. Assess sputum cultures.
c. Make sure IV is patent.
d. Force fluids.

a. Calcium 12 mg/dL

A client is prescribed calcipotriene (Dovonex) for treatment of psoriasis. Which assessment finding requires immediate intervention by the nurse?
a. Calcium 12 mg/dL
b. Potassium 3.8 meq/L
c. Sodium 135 mmol/L
d. Phosphorus 2.5 mg/dL

d. Ask client if he or she has any allergies.

Before applying povidone-iodine (Betadine) to a client's skin, what is a primary nursing intervention?
a. Apply a cortisone cream.
b. Wash the skin.
c. Shave and prepare the area.
d. Ask client if he or she has any allergies.

b. Call the health care provider if you have muscle weakness.

A client is prescribed isotretinoin (Accutane). What is the most important instruction to teach the client before beginning this medication?
a. Do not go out in the sun while on this medication.
b. Call the health care provider if you have muscle weakness

b. Avoid driving a motor vehicle until stabilized on the drug.

A client tells the nurse that he has started to take an OTC antihistamine, diphenhydramine. In teaching him about side effects, what is most important for the nurse to tell the client?
a. Do not to take this drug at bedtime to avoid insomnia.
b. Avoid dri

c. Acute pharyngitis.

The client complains of a sore throat and has been told it is due to beta-hemolytic streptococcal infection. The nurse realizes this condition is called what?
a. Acute rhinitis.
b. Acute sinusitis.
c. Acute pharyngitis.
d. Acute rhinorrhea.

d. Limit the drug to 5 days of use to prevent rebound nasal congestion.

A client is prescribed the decongestant oxymetazoline (Afrin) nasal spray. What should the nurse teach the client?
a. Take this drug at bedtime as a sleep aid.
b. Directly spray away from the nasal septum and gently sniff.
c. This drug may be used in main

b. To loosen bronchial secretions so they can be eliminated by coughing

A client has been prescribed guaifenesin (Robitussin). The nurse realizes that the purpose of the drug is to accomplish what?
a. To treat allergic rhinitis and prevent motion sickness
b. To loosen bronchial secretions so they can be eliminated by coughing

d. Dry nasal mucosa

Beclomethasone (Beconase) has been prescribed for a client with allergic rhinitis. The nurse teaches the client that which is the most common side effect from continuous use?
a. Dizziness
b. Rhinorrhea
c. Hallucinations
d. Dry nasal mucosa

a. Take medication with food to decrease gastric distress.
b. Avoid alcohol and other central nervous system depressants.
c. Notify the health care provider if confusion or hypotension occurs.
d. Take sugarless candy, gum, or ice chips for temporary relie

The nurse is teaching a client about diphenhydramine (Benadryl). Which are topics to include? (Select all that apply.)
a. Take medication with food to decrease gastric distress.
b. Avoid alcohol and other central nervous system depressants.
c. Notify the

a. "Do not drive after taking this medication.

The nurse is caring for a client who is taking a first-generation antihistamine. What is the most important fact for the nurse to teach the client?
a. "Do not drive after taking this medication."
b. "Make sure you drink a lot of liquids while on this medi

d. "You may be able to safely take a second-generation antihistamine.

The nurse is caring for a client in the clinic who states that he is afraid of taking antihistamines because he is a truck driver. What is the best information for the nurse to give this client?
a. "Take the medication only when you are not driving."
b.

Administer guaifenesin.

The client tells the nurse that she has a bad cold, is coughing, and feels like she has "stuff" in her lungs. What should the nurse do?
a. Administer dextromethorphan.
b. Administer guaifenesin.
c. Encourage the client to drink fluids hourly.
d. Administe

b. This medication has fewer sedative effects.

What is the most important thing for the nurse to teach a client who is switching allergy medications from diphenhydramine (Benadryl) to loratadine (Claritin)?
a. This medication can potentially cause dysrhythmias.
b. This medication has fewer sedative ef

b. "Overuse of nasal decongestants results in rebound congestion.

A client complains of worsening nasal congestion despite the use of oxymetazoline (Afrin) nasal spray every 2 hours. What is the nurse's best response?
a. "Oxymetazoline is not an effective nasal decongestant."
b. "Overuse of nasal decongestants results i

b. "This medication will help prevent the inflammatory response of my allergies.

Which statement indicates that the client understands the teaching about beclomethasone diproprionate (Beconase)?
a. "I will need to taper off the medication to prevent acute adrenal crisis."
b. "This medication will help prevent the inflammatory response

a. "This medication may cause drowsiness and dizziness.

A client is prescribed an antitussive medication. What is the most important thing for the nurse to teach the client?
a. "This medication may cause drowsiness and dizziness."
b. "Watch out for diarrhea and abdominal cramping."
c. "This may cause tremors a

c. Increase fluid intake in order to decrease viscosity of secretions.

Which is the best instruction for the nurse to include when teaching a client about the use of expectorants?
a. Restrict fluids in order to decrease mucus production.
b. Take the medication once a day only, at bedtime.
c. Increase fluid intake in order to

a. Asthma

A client is diagnosed with a pulmonary disorder that causes COPD. Lungs tissue changes are normally reversible with this condition. The nurse understands that which is the client's most likely diagnosis?
a. Asthma
b. Emphysema
c. Bronchiectasis
d. Chronic

b. epinephrine (Adrenalin)

A client with COPD has an acute bronchospasm. The nurse knows that which is the best medication for this emergency situation?
a. zafirlukast (Accolate)
b. epinephrine (Adrenalin)
c. dexamethasone (Decadron)
d. oxtriphylline-theophyllinate (Choledyl)

c. Increased heart rate

A client is taking aminophylline-theophylline ethylenediamine (Somophyllin). For what should the nurse monitor the client?
a. Drowsiness
b. Hypoglycemia
c. Increased heart rate
d. Decreased white blood cell count

b. 10 to 20 mcg/mL

A client is prescribed theophylline to relax the smooth muscles of the bronchi. The nurse monitors the client's theophylline serum levels to maintain which therapeutic range?
a. 1 to 10 mcg/mL
b. 10 to 20 mcg/mL
c. 20 to 30 mcg/mL
d. 30 to 40 mcg/mL

a. Maintenance treatment of asthma

A client with COPD is taking a leukotriene antagonist, montelukast (Singulair). The nurse is aware that this medication is given for which purpose?
a. Maintenance treatment of asthma
b. Treatment of an acute asthma attack
c. Reversing bronchospasm associa

c. Continue to assess the client's oxygenation.

The nurse is caring for a client with a theophylline level of 14 mcg/mL. What is the priority nursing intervention?
a. Increase the IV drip rate.
b. Monitor the client for toxicity.
c. Continue to assess the client's oxygenation.
d. Stop the IV for an hou

d. Tachycardia

Discharge teaching to a client receiving a beta-agonist bronchodilator should emphasize reporting which side effect?
a. Hypoglycemia
b. Nonproductive cough
c. Sedation
d. Tachycardia

d. St. John's wort

The nurse instructs the client to avoid which over-the-counter products when taking theophylline (Theo-Dur)?
a. acetaminophen (Tylenol)
b. echinacea
c. diphenhydramine (Benadryl)
d. St. John's wort

a. Monitor client for potential chest pain.

A nurse reviews a client's medication history and notes that the client is taking a nonselective adrenergic agonist bronchodilator and has a history of coronary artery disease. What is a priority nursing intervention?
a. Monitor client for potential chest

d. Salmeterol has a longer duration of action.

The nurse is instructing a client about the advantages of salmeterol (Serevent) over other beta2 agonists such as albuterol (Proventil). How will the nurse explain to the client the difference in these two medications?
a. Salmeterol has a shorter onset of

c. "This medication will prevent the inflammation that causes your asthma attack.

Client teaching regarding the use of antileukotriene agents such as zafirlukast (Accolate) should include which statement?
a. "Take the medication as soon as you begin wheezing."
b. "It will take about 3 weeks before you notice a therapeutic effect."
c.

c. Administer a beta2 adrenergic agonist.

A client with a history of asthma is short of breath and says, "I feel like I'm having an asthmatic attack." What is the nurse's best action?
a. Call a code.
b. Ask the client to describe the symptoms.
c. Administer a beta2 adrenergic agonist.
d. Administ

a. Monitor for heart rate >100 beats/min.

A client has taken metaproterenol. What is the nurse's priority action?
a. Monitor for heart rate >100 beats/min.
b. Tell the client not to drive for 2 hours.
c. Monitor for sedation.
d. Assess for elevated blood pressure.

b. Rinse his mouth with water after each use.

A client demonstrates understanding of flunisolide (AeroBid) by saying that he will do what?
a. Take two puffs to treat an acute asthma attack.
b. Rinse his mouth with water after each use.
c. Immediately stop taking his oral prednisone when he starts usi

d. Teach the child to use a spacer.

The nurse is caring for a young child who has been prescribed an inhaler for control of her asthma. The child is having difficulty using the inhaler. What is the nurse's best action?
a. Tell the parent to hold the inhaler for the child.
b. Ask the health

d. The client with atrial fibrillation with a rate of 100

The nurse is caring for clients on the pulmonary unit. Which client should not receive epinephrine if ordered?
a. The client with a history of emphysema
b. The client with a history of type 2 diabetes
c. The client who is 16 years old
d. The client with a

b. Administer the albuterol first, wait 5 minutes, and administer ipratropium bromide, followed by beclomethasone several minutes later.

The health care provider orders ipratropium bromide (Atrovent), albuterol (Proventil), and beclomethasone (Vanceril) inhalers for a client. What is the nurse's best action?
a. Question the order; three inhalers should not be given at one time.
b. Administ

c. "Hold your breath for 10 seconds if you can after you inhale the medication.

Which instruction will the nurse include when teaching a client about the proper use of metered-dose inhalers?
a. "After you inhale the medication once, repeat until you obtain relief."
b. "Make sure that you puff out air repeatedly after you inhale the m

c. Liquefying and loosening of bronchial secretions

What will the nurse expect to find that would indicate a therapeutic effect of acetylcysteine (Mucomyst)?
a. Decreased cough reflex
b. Decreased nasal secretions
c. Liquefying and loosening of bronchial secretions
d. Relief of bronchospasms

c. Monitor blood glucose levels every 4 hours when taking albuterol.

What is the most important thing for the nurse to teach the client with a history of diabetes and asthma who has started on albuterol PRN?
a. Take Tylenol for headaches when taking albuterol.
b. Monitor for orthostatic hypotension every 2 hours when takin

b. "Take the ipratropium at least 5 minutes before the cromolyn.

A client is prescribed ipratropium and cromolyn sodium. What will the nurse teach the client?
a. "Do not take these medications within 4 hours of each other."
b. "Take the ipratropium at least 5 minutes before the cromolyn."
c. "Administer both medication

c. Hold the next dose of theophylline.

A client taking an oral theophylline preparation is due for her next dose and has a blood pressure of 100/50 mm Hg and a heart rate of 110. The client is irritable. What is the best action for the nurse to take?
a. Continue to monitor the client.
b. Call

b. Lack of exercise

A client complains of constipation and requires a laxative. In providing teaching to the client, the nurse reviews the common causes of constipation, including which cause?
a. Motion sickness
b. Lack of exercise
c. Food intolerance
d. Bacteria (Escherichi

b. Block serotonin receptors in the CTZ

A client has nausea and is taking ondansetron (Zofran). The nurse explains that the action of this drug is what?
a. Stimulate the CTZ
b. Block serotonin receptors in the CTZ
c. Block dopamine receptors in the CTZ
d. Coat the wall of the GI tract and absor

a. Acts on smooth intestinal muscle to gently increase peristalsis

A client who has constipation is prescribed a bisacodyl suppository. The nurse explains that bisacodyl does what?
a. Acts on smooth intestinal muscle to gently increase peristalsis
b. Absorbs water into the intestines to increase bulk and peristalsis
c. L

d. Dry mouth

A client is using the scopolamine patch to prevent motion sickness. The nurse teaches the client that which is a common side effect of this drug?
a. Diarrhea
b. Vomiting
c. Insomnia
d. Dry mouth

c. Alcohol

When metoclopramide (Raglan) is given for nausea, the client is cautioned to avoid which substance?
a. Milk
b. MAOIs
c. Alcohol
d. Carbonated beverages

a. Warn the client to avoid laxative abuse.
b. Record the frequency of bowel movements.
c. Warn the client against taking sedatives concurrently.
d. Encourage the client to increase fluids.
e. Instruct the client to avoid this drug if he or she has narrow

The nurse is administering opium tincture (paregoric) to a client. Which should be included in the client teaching regarding this medication? (Select all that apply.)
a. Warn the client to avoid laxative abuse.
b. Record the frequency of bowel movements.

a. Client has not had a bowel movement in 3 days.

Which assessment finding will need intervention and is related to the client's use of aluminum hydroxide (Amphojel)?
a. Client has not had a bowel movement in 3 days.
b. Client has had one loose stool this week.
c. Client is complaining of gastric upset.

c. Client taking magnesium-containing antacids who has renal failure.

Which client needs immediate intervention?
a. Client taking aluminum-containing antacids with complaints of reflux.
b. Client taking calcium-containing antacids who is hypocalcemic.
c. Client taking magnesium-containing antacids who has renal failure.
d.

a. Assess for metabolic alkalosis.

What assessment has the highest priority for a client using sodium bicarbonate to treat gastric hyperacidity?
a. Assess for metabolic alkalosis.
b. Assess for fluid volume deficit.
c. Assess for hyperkalemia.
d. Assess for hypercalcemia.

b. Potential risk for bleeding related to thrombocytopenia

Which nursing diagnoses is appropriate for a client receiving famotidine (Pepcid)?
a. Increased risk for infection related to immunosuppression
b. Potential risk for bleeding related to thrombocytopenia
c. Alteration in urinary elimination related to rete

b. "Smoking decreases the effects of this medication, so I should look into cessation programs.

Which statement demonstrates to the nurse that the client understands instructions regarding the use of histamine2-receptor antagonists?
a. "Since I am taking this medication, it is all right for me to eat spicy foods."
b. "Smoking decreases the effects o

d. Administer the medications and assess the client for relief.

A client is prescribed Lorazepam (Ativan) and a glucocorticoid during chemotherapy treatments. What is the nurse's best action?
a. Call the health care provider and question the order.
b. Only administer the Ativan if the client seems anxious.
c. Administ

d. pantoprazole (Protonix)

A nurse is caring for a client who is unable to tolerate oral medications. The nurse anticipates that the client may be prescribed which proton pump inhibitor to be administered intravenously?
a. esomeprazole (Nexium)
b. lansoprazole (Prevacid)
c. omepraz

c. "I will apply the scopolamine patches to rotating sites on my arms.

Which client statement indicates that further teaching is needed?
a. "I will not drive while I am taking these medications because they may cause drowsiness."
b. "I may take Tylenol to treat the headache caused by ondansetron (Zofran)."
c. "I will apply t

b. Gastric assessment

The nurse is administering loperamide (Imodium) to a client with diarrhea. What assessment is essential for this client?
a. Vascular assessment
b. Gastric assessment
c. Hourly blood pressure measurements
d. White blood count

c. Decrease in gastric motility

Which outcome assessment is essential to monitor for the client taking diphenoxylate (Lomotil)?
a. Increase in bowel sounds
b. Increase in number of bowel movements
c. Decrease in gastric motility
d. Decrease in urination

c. Administer 30 minutes before meals and at bedtime.

The nurse is planning to administer metoclopramide (Reglan). What is a primary intervention?
a. Administer with food to decrease gastrointestinal upset.
b. Administer every 6 hours around the clock.
c. Administer 30 minutes before meals and at bedtime.
d.

c. Combination therapy blocks different vomiting pathways.

What will the nurse teach the client about the reason for administering multiple medications for relief of nausea and vomiting?
a. Combination therapy decreases the risk of constipation.
b. Combination therapy is more cost-effective.
c. Combination therap

b. Fluid volume deficit related to nausea and vomiting

In developing a plan of care for a client receiving an antihistamine antiemetic agent, which nursing diagnosis would be of highest priority?
a. Knowledge deficit regarding medication administration
b. Fluid volume deficit related to nausea and vomiting
c.

c. "Brush your teeth and gargle to help with dryness in your mouth.

What instruction is most important for the nurse to teach a client who is taking an anticholinergic agent to treat nausea and vomiting?
a. "Assess your stools for dark streaks."
b. "Do not take more than two doses of this medication."
c. "Brush your teeth

b. Weigh the client before chemotherapy.

A client is prescribed granisetron (Kytril) IV for relief of nausea and vomiting caused by cancer chemotherapy. What intervention is most appropriate for this client?
a. Administer the medication at least 12 hours before the start of chemotherapy.
b. Weig

b. Administer ondansetron HCL (Zofran) 30 minutes before therapy and two doses after therapy.

A client is starting cisplatin therapy for cancer. What intervention is appropriate for this client?
a. Administer granisetron (Kytril) 60 minutes before therapy and for several days after surgery.
b. Administer ondansetron HCL (Zofran) 30 minutes before

c. Evaluate renal function.

Before administering a stimulant laxative to a client, which nursing intervention is the priority?
a. Obtain a history of constipation and causes.
b. Record baseline vital signs.
c. Evaluate renal function.
d. Assess fluid and electrolyte balance.

c. Monitor signs and symptoms of fluid and electrolyte imbalance.

Which assessment is most important for the client who is taking stimulant laxatives?
a. Monitor bowel elimination daily.
b. Monitor intake and output.
c. Monitor signs and symptoms of fluid and electrolyte imbalance.
d. Monitor heart rate and blood pressu

d. "After 3 days, switch patch to alternate ear."
e. "Apply patch 4 hours before effect is desired."
f. "Drowsiness is a concern while on this medication.

A client is prescribed scopolamine. What information will the nurse include on the teaching plan for this client? (Select all that apply.)
a. "Do not take this medication if you are dizzy."
b. "Do not use laxatives while on this medication."
c. "Do not us

a. Helicobacter pylori

A client is diagnosed with peptic ulcer disease. The nurse realizes that which factor is a predisposing factor for this condition?
a. Helicobacter pylori
b. hyposecretion of pepsin
c. decreased hydrochloric acid
d. decreased number of parietal cells

d. To combine with protein to form a viscous substance that forms a protective covering of ulcer

When a client is given sucralfate (Carafate), the nurse knows that its mode of action is what?
a. To neutralize gastric acidity
b. To inhibit gastric acid secretion by inhibiting histamine at H2 receptors in parietal cells
c. To suppress gastric acid secr

c. The drug must be administered separate from an antacid by at least 1 hour
e. Smoking should be avoided while taking this drug
f. Foods high in vitamin B12 should be increased in diet

A client is taking ranitidine (Zantac). The nurse who is teaching the client about this drug should include which information? (Select all that apply.)
a. Drug-induced impotence is irreversible
b. The drug must be administered 30 minutes before meals
c. T

d. Antacids neutralize HCl and reduce pepsin activity.

When a client complains of pain accompanying a peptic ulcer, why should an antacid be given?
a. Antacids decrease GI motility.
b. Antacids decrease gastric acid secretion.
c. Aluminum hydroxide is a systemic antacid.
d. Antacids neutralize HCl and reduce

b. Dizziness
d. Headaches
f. Decreased libido

A client is taking famotidine (Pepcid) to inhibit gastric secretions. What are the side effects of famotidine? (Select all that apply.)
a. Diarrhea
b. Dizziness
c. Dry mouth
d. Headaches
e. Blurred vision
f. Decreased libido

b. "I will drink 2 ounces of water after taking aluminum hydroxide.

A client has just been prescribed aluminum hydroxide (Amphojel, ALternaGEL, Alu-Tab) for peptic ulcer pain. The nurse has provided instructions to the client. Which statement by the client indicates to the nurse that the client understands the instruction

a. Administer just before meals.

What is a priority nursing intervention when administering ranitidine (Zantac)?
a. Administer just before meals.
b. Administer right after eating.
c. Administer 1 to 2 hours after meals.
d. Administer during meals.

c. The client has no throat pain.

The health care provider prescribes lansoprazole (Prevacid) to a client. Which assessment indicates to the nurse that the medication has had a therapeutic effect?
a. The client has no diarrhea.
b. The client has no gastric pain.
c. The client has no throa

b. Absent bowel sounds, hard abdomen

The nurse is caring for a client who is taking sucralfate (Carafate, Sulcrate) for treatment of a duodenal ulcer. Which assessment requires action by the nurse?
a. Sodium level 140 mEq/L
b. Absent bowel sounds, hard abdomen
c. Urinary output 30 mL/hr
d. C

c. Allow the tablet to dissolve in water before administering.

When administering sucralfate (Carafate) to a client with a nasogastric tube, what is an essential intervention?
a. Crush the tablet into a fine powder before mixing it with water.
b. Administer with a bolus tube feeding.
c. Allow the tablet to dissolve i

c. "This medication will form a protective barrier over the gastric mucosa.

What information should the nurse include in a teaching plan for the client who is prescribed sucralfate (Carafate)?
a. "This medication will neutralize gastric acid."
b. "This medication will enhance gastric absorption of meals."
c. "This medication will

b. Administer misoprostol.
d. Instruct the client to take omeprazole with the aspirin.

The nurse is caring for a client who is experiencing gastric distress from the long-term use of aspirin for treatment of arthritis. What is the best intervention for this client? (Select all that apply.)
a. Stop all aspirin therapy.
b. Administer misopros

a. Dehydration

The nurse reviews the client's list of medication, which includes mannitol. The nurse must be aware that which condition is a contraindication for use of this drug?
a. Dehydration
b. Kidney stones
c. Eczema
d. Gout

c. Cyclopentolate

The client is being prepared for an eye examination. When the nurse takes the health history, the client says that she is sensitive to atropine sulfate. What drug might be used instead for the examination?
a. Diclofenac
b. Suprofen
c. Cyclopentolate
d. Be

c. Electrolytes

An 85-year-old client is taking acetazolamide, a carbonic anhydrase inhibitor. A nursing intervention associated with clients receiving this drug is to monitor what?
a. Weight
b. Complete blood count
c. Electrolytes
d. Urine output

d. travoprost

The nurse reviews the African-American client's list of medications. It is important for the nurse to be aware that the prostaglandin analogue more effective in African Americans than in non-African Americans is wha?
a. latanoprost
b. bimatoprost
c. unopr

a. School-aged children may need only one drug, not a combination.

The school nurse is preparing a presentation for the parent-teacher association meeting on medications commonly used in school-aged children. It is important to note what primary disadvantage of the use of combination products such as Cortisporin Otic?
a.

a. Hydrogen peroxide

The camp nurse reviews the "shopping list" of supplies needed for the upcoming camping season. What product is recommended to prevent and treat chronic impaction of cerumen?
a. Hydrogen peroxide
b. Rubbing alcohol
c. Charcoal
d. Salt solution

a. Instruct the client to report changes in vision and breathing.
b. Maintain sterile technique and prevent dropper

The nurse prepares a health teaching plan for the client with glaucoma. Which important nursing intervention are included for this client? (Select all that apply.)
a. Instruct the client to report changes in vision and breathing.
b. Maintain sterile techn

d. Client's pupils are constricted to 2 mm.

The nurse administers pilocarpine (Pilocar) to a client with glaucoma. Which assessment finding would indicate a therapeutic effect of the medication?
a. Client's eyes appear clear, without drainage.
b. Client states that her eyes feel very dry.
c. Client

c. "I should rinse the eye dropper with tap water after each use.

Which statement, made by a client, indicates to the nurse a need for further client teaching regarding proper administration of eye drops?
a. "I will put pressure on the inside corner of my eye after I administer the drops."
b. "I will be careful not to t

b. Warm the eardrops to room temperature before administration.

The nurse is planning to administer eardrops. Which intervention is essential to include in the plan of care?
a. Eardrops should be cool when being administered.
b. Warm the eardrops to room temperature before administration.
c. The pinna of an adult shou

b. carbamide peroxide

A client is complaining of excessive earwax that diminishes hearing ability. What medication will the nurse use to assist the client?
a. acetic acid
b. carbamide peroxide
c. hydrocortisone
d. glycerin

b. Instruct the client that one drop is optimal.

The nurse evaluates the client using eyedrops. The client puts two drops into his eye. What is the nurse's best action?
a. Continue to observe the client.
b. Instruct the client that one drop is optimal.
c. Have the client irrigate his eye to remove exces

b. A horny layer of epidermis

The nurse reviews the client's list of medications and recalls that the purpose of keratolytic agents is to remove what?
a. A horny layer of dermis
b. A horny layer of epidermis
c. Erythematous lesions
d. Hair follicles

a. Avoid sunlight.
c. Monitor CBC, glucose, and lipids.
d. Do not breastfeed or give blood.

Nursing implications for health teaching with clients taking isotretinoin include which implications? (Select all that apply.)
a. Avoid sunlight.
b. Monitor weight
c. Monitor CBC, glucose, and lipids.
d. Do not breastfeed or give blood.

c. Assess lesions

The nurse is doing health teaching with a client with psoriasis. Which is a nursing implication of the new biologic agents for the management of psoriasis?
a. Daily weight
b. Monitor electrolytes
c. Assess lesions
d. Monitor CBC and T-cell count

d. finasteride

A 55-year-old man has a chief complaint: "I'm going bald." Which drug is used to treat male pattern baldness?
a. dexamethasone
b. PABA
c. minoxidil
d. finasteride

a. Metabolic acidosis
c. Respiratory alkalosis

The client has second- and third-degree burns over 25% of his body. Mafenide acetate has been ordered. What acid-base imbalance can result from its use? (Select all that apply.)
a. Metabolic acidosis
b. Metabolic alkalosis
c. Respiratory alkalosis
d. Resp

b. Thinning of the skin
d. Purpura

The nurse reviews the client's medication history. Based on the client's prolonged use of glucocorticoids, what does the assessment include? (Select all that apply.)
a. Obesity
b. Thinning of the skin
c. Erythematous lesions
d. Purpura

b. A review of iPLEDGE educational materials
c. That a negative pregnancy test is required before each monthly refill

A 20-year-old woman comes to the clinic for follow-up related to isotretinoin use. The nurse reviews the iPLEDGE program, which includes which important information? (Select all that apply.)
a. That an effective method of contraception must be used throug

a. Sunscreen products should contain information about UVA and UVB SPF protection.
b. UVB radiation is greatest between 10 AM and 4 PM.
d. SPF should be at least 15 in sunscreen products.

The school nurse prepares a program for junior high school students on sun safety. What is important information to include? (Select all that apply.)
a. Sunscreen products should contain information about UVA and UVB SPF protection.
b. UVB radiation is gr

d. Silver sulfadiazine cream

Which intervention is most appropriate for the client with second-degree burns?
a. IV antibiotics
b. Isolation
c. IV dextrose infusion
d. Silver sulfadiazine cream

a. Perform pregnancy test.

A 20-year-old client is starting isotretinoin (Accutane) therapy. What is an essential nursing intervention for this client?
a. Perform pregnancy test.
b. Assess sputum cultures.
c. Make sure IV is patent.
d. Force fluids.

a. Calcium 12 mg/dL

A client is prescribed calcipotriene (Dovonex) for treatment of psoriasis. Which assessment finding requires immediate intervention by the nurse?
a. Calcium 12 mg/dL
b. Potassium 3.8 meq/L
c. Sodium 135 mmol/L
d. Phosphorus 2.5 mg/dL

d. Ask client if he or she has any allergies.

Before applying povidone-iodine (Betadine) to a client's skin, what is a primary nursing intervention?
a. Apply a cortisone cream.
b. Wash the skin.
c. Shave and prepare the area.
d. Ask client if he or she has any allergies.

b. Call the health care provider if you have muscle weakness.

A client is prescribed isotretinoin (Accutane). What is the most important instruction to teach the client before beginning this medication?
a. Do not go out in the sun while on this medication.
b. Call the health care provider if you have muscle weakness

The nurse is having difficulty reading a physician's order for a medication. The nurse knows the physician is very busy and does not like to be called. The nurse should

Call the physician to have the order clarified

The client has an order for 2 tablespoons of Milk of Magnesia. The nurse converts this dose to the metric system and gives the client:

30 mL

Most medication errors occur when the nurse:

Fails to follow routine procedures

A client is to receive cephalexin (Keflex) 500 mg PO. The pharmacy has sent 250-mg tablets. The nurse gives:

2 tablets

When identifying a new client before administering medications, the nurse asks the client to state his name. The client does not state the correct name. The nurse asks again, and the client states still another name. What is the nurse's next action?

Investigate the client's mental status before administering any further medications.

A client is transitioning from the hospital to the home environment. A home care referral is obtained. What is a priority, in relation to safe medication administration, for the discharge nurse?

Ensure the home care agency is aware of medication and health teaching needs.

A nursing student takes a client's antibiotic to his room. The client asks the nursing student what it is and why he should take it. The nursing student's reply includes the following information:

The name of the medication and a description of its desired effect

The nurse is administering a sustained-release capsule to a new client. The client insists that he cannot swallow pills. The best course of action for the nurse is to:

Ask the physician to change the order

The nurse takes a medication to a client, and the client tells the nurse to take it away because she is not going to take it. The nurse's first action should be to:

Ask the client's reason for refusal

The nurse selects the route for administering medication according to:

The prescriber's orders

A client is receiving an IV push medication. If this type of drug infiltrates into the outer tissues, the nurse will:

Follow facility policy or drug manufacturer's directions

If a client who is receiving IV fluids develops tenderness, warmth, erythema, and pain at the site, the nurse suspects:

Phlebitis

A nurse administering medications has many responsibilities. Among these responsibilities is a knowledge of pharmacokinetics. Which statement is the best description of pharmacokinetics?

The study of how medications enter the body, reach their site of action, metabolize, and exit the bod

The following orders were written by a prescriber (physician, advanced practice nurse, physician's assistant). Which order is written correctly?

CORRECT ANSWER IS C
A) Aspirin 2 tablets prn
B) Haloperidol (Haldol) � tablet at bedtime
C) Zolpidem (Ambien) 5 mg PO at bedtime prn
D) Levothyroxine (Synthroid) 0.05 mg 1 tablet

To better control the client's blood glucose level, the physician orders a high regular insulin dosage of 20 units of U-500 insulin. The nurse has only a U-100 syringe. How many units will be given

4
U-500 insulin is 5 times as strong as U-100 insulin. Therefore the amount of U-500 insulin should be divided by 5; 20 units � 5 = 4 units.

The nurse is administering an intramuscular (IM) injection. The Z-track method is recommended for IM injections because:

It minimizes local skin irritation by sealing the medication in muscle tissue.

What is the best nursing practice for administrating a controlled substance if part of the medication must be discarded?

The nurse documents on the medication administration record and the control inventory form, and has a second nurse witness the medication being discarded.

When administering medications, it is essential for the nurse to have an understanding of basic arithmetic to calculate doses. The physician has ordered 250 mg of a medication that is available in 1-g amount. The vial reads 2 ml = 1 g. What dose would be

0.5 ml = 250 mg of this medication.
(Dose ordered/dose on hand) � amount on hand = amount administered
[250 mg/1000 mg (1 g)] � 2 ml = 500/1000 = � ml or, in decimals, 0.5 ml

While the nurse is administering medication, the client says, "This pill looks different from what I usually take." What is the nurse's best action?

Go recheck the medication order, taking along the medication.

The client is a 40-year-old man who weighs 160 lb and is 5 feet 9 inches tall. The order is for 5 ml of a medication to be given as a deep intramuscular (IM) injection. What size of syringe and gauge and length of needle should the nurse use for best prac

Two 3-ml syringes, 20- to 23-gauge, 1�-inch needle

A site that was a traditional location for intramuscular (IM) injections in the past is no longer recommended because its use carries the risk of striking the underlying sciatic nerve or major blood vessel. What is the name of this site?

Dorsogluteal

What medication interaction can cause profound hypotension if taken with Nitro?

Viagra
Cialis

Can patients take Nitro 24 hours a day?

No. They must maintain a 6 to 8 hour nitrate-free period every 24 hours after acute episode to avoid tolerance.

WHat lab value should be monitored for patients taken Warfarin?

INR and Prothrombin time

What is the antidote for warfarin?

Vitamin K

Distinguish between low molecular weight heparin and unfractionated heparin?

LMWH action: blocks action of Factors Xa and Ila without appreciably affecting thrombin or prothrombin
Un-fractionated heparin: inhibits conversion of prothrombin to thrombin thus preventing fibrin formation

What is perhaps the most dangerous complication of unfractionated heparin therapy?

Life-threatening thrombocytopenia

What is the antidote for beta-blockers?

glucagon

What is the antidote for un-fractionated heparin?

protamine sulfate

To what medication class does Procainamide belong to? What foods should be avoided while taking this medication?

Na-channel blocking agents. These drugs are "antidysrhythmics"
AVOID citrus juices, antacids, and milk products

What is amnioderone?

A K-CHANNEL blocking agent, an antidysrhythmic medication

What are your most common ACE Inhibitors? What is the most common side effect of these medications? How should the pills be taken?

Enalapril & Lisinopril
SE: cough
Take on an empty stomach 1 hour prior to a meal or 2 hours after a meal

What are perhaps the most common side effects of anticholinergics?

dry mouth, constipation, and blurred vision,

What is hyrdALAZINE (Apresoline)??

a "centrally acting vasodilator"
directly relaxes arteriolar vascular smooth muscle resulting in lowered peripheral vascular resistance and reflex tachycardia

Why would you take a bile acid sequestrant agent? Name an example.

To increase lipid excretion in stool, and thus lower serum lipids.
ex: Questran

When should the nurse know to hold a dose of Digoxin?

If the patient's HR is <60bpm

What is the therapeutic range for Digoxin?

therapeutic range 0.8 - 2 ng /mL (toxic: greater than 2.4 ng/mL)

What are some examples of thrombolytic agents? What problems are these used for?

Streptokinase
Urokinase
Used in case of clots associated with :
myocardial infarction
deep venous thrombosis
pulmonary embolism
thrombosed intravenous catheters

How does Robitussin work?
How should you give it?

It liquifies secretions in the respiratory tract by decreasing the surface tension.
Give with plenty of water, and tell the pt. to avoid dairy and caffeine while On the med.

What is one of the major contraindications to Isoinazid therapy for TB?
What foods should patients avoid on this drug?

acute liver dysfunction
Avoid foods with a) histamine (sauerkraut, jack-tuna, yeast)
b) tyramine (aged cheese, cured meat, smoked fish)

What is Ethambutol prescribed for and what is a danger of taking this med?

It is a drug prescribed in combination with Isoinazid and Rifampin (and Streptomycin) for TB treatment. It can adversely affect the eyes, and patients on this med should get regular eye exams.

What type of diet should patients on a glucocorticoid be on?

Low-sodium, possibly calorie restricted.

What suffix is typically associated with 2nd generation antihistamines?

adine", for example:
loratadine (Claritin)
fexofenadine (Allegra)
desloratadine (Clarinex)
cetirizine (ZyrTEC)

What is cloNIDine (Catapres) ?

An anti-hypertensive

Define hemiparesis

weakness on one side of the body. It is less severe than hemiplegia � the total paralysis of the arm, leg, and trunk on one side of the body. Thus, the patient can move the impaired side of his body, but with reduced muscular strength.

Why would you position a client in left trendelenberg position if you suspected an air emboli?

Because it traps the air in the right side of the heart

What is the antidote to Heparin?

Protamine sulfate

What is Propecia typically prescribed for?

Allopecia
Benign Prostatic Hypertrophy Agent

What are some of the serious side effects of anti-seizure medications that should be reported

unsteady gait, slurred speech, extreme fatigue, blurred vision, or feelings of suicide.
Increased hunger, increased thirst, or increased urination are additional serious side effects.

What lab values, following overdose with Tylenol would the nurse expect to follow closely?

Those that indicate liver damage, such as Bilirubin, ALT, and AST.

What drug interaction is important to know about Lithium Carbonate?

DO NOT give Lithium with diuretics, as it can cause the kidney to retain too much lithium, resulting in subsequent toxicity.

What is a 'dystonic reaction' and what drug class does this sometimes occur with?

symptoms of dystonia include stiffness and muscle rigidity, difficulty speaking, internal agitation and may progress to oculogyric crisis and hypertensive crisis. Typical antipsychotic drugs can cause this, such as Haldolperidol.

How long does it take Insulin Lispro (Humalog) to begin acting?

15 minutes; peak 30-90 minutes; duration of activity is less than 5 hours

When do the primitive reflexes like the tonic neck, Moro, and palmar grasp reflexes disappear in newborn development?

6 months

What is Ewing's sarcoma and what age group does it usually affect? How do you treat it?

a rare disease in which cancer cells are found in the bone or in soft tissue. The most common areas in which it occurs are the pelvis, the femur, the humerus, the ribs and clavicle (collar bone). Usually treated with multi-drug chemo. Ewing's sarcoma occu

What is diplopia?

double vision

What is Fluphenazine (Prolixin)

An antipsychotic medication

If a patient has thrombophlebitis on one leg, what should your immediate action be for the leg?

Elevate it on a pillow

Why is Aluminum hydroxide (Amphojel) given?

Aluminum binds phosphates that tend to accumulate in the client with chronic renal failure due to decreased filtration capacity of the kidney. Antacids such as Amphojel or Basogel are commonly used to accomplish a decreased serum phosphate.

When is the level of forced expiratory volume of concern?

A forced expiratory volume is of a concern if it is 50% or less.

Where would the nurse palpate the uterine fundus during the period 1-24 hours following delivery?

The uterus should be felt at the level of the umbilicus from about 1 to 24 hours after birth. The fundus (top of the uterus) will fall approximately 1 centimeter (or 1 fingerbreadth) each day for the next 10 days.

What is keratitis and how do you prevent it?

Keratitis is eye inflammation from a corneal ulcer or abrasion. Keratitis is caused by exposure to the air without the normal blink. It requires regular applications of moisturizing ointment to the exposed cornea and a plastic bubble shield or eye patch.

When would you instill saline for tracheostomy suctioning?

According to evidence-based practice, the use of saline is no longer recommended during routine suctioning. However, if a client is suspected to have a mucous plug in the larger bronchials or in an artificial airway (such as a tracheostomy tube), the nurs

What are the symptoms of seratonin syndrome?

**confusion, nausea, palpitations, increased muscle tone with twitching muscles, and agitation.
(The most common drug combinations associated with serotonin syndrome involve the MAOIs, SSRIs, and the tricyclic antidepressants)

HOw long after surgery can a gastronomy tube be used to instill feedings?

After surgery for gastrostomy tube placement, the catheter is left open and attached to gravity drainage of air & stomach contents for 24 hours or more.

What ECG change is suggestive of hyperkalemia?

A tall peaked T wave is a finding in hyperkalemia.

What is trigeminal neuralgia?

a nerve disorder that causes a stabbing or electric-shock-like pain in parts of the face.

In order to prevent aspirin toxicity, what lab value should be monitored?

Serum albumin. When highly protein-bound drugs are administered to clients with low serum albumin (protein) levels, excess free (unbound) drug can cause exaggerated and dangerous effects. Aspirin is a protein-bound drug.

How long should a client with GERD sit upright following a meal? How long before going to bed should the client eat?

Sit upright 2 hours post-meal (allows the stomach to empty)
don't eat 2 hours prior to bedtime

Why do newborns born to diabetic mothers sometimes experience hypoglycemia?

After delivery, high glucose levels which crossed the placenta to the fetus are suddenly stopped. The newborn continues to secrete insulin in anticipation of the glucose. When oral feedings begin, the newborn will adjust insulin production within a day or

What is tylenol #3?

Tylenol + Codeine

How many expiratory efforts should the client using a peak flow meter put forth, and which reading do they record?

Three times, record the highest reading of the three.

How exactly does Vitamin C interfere with a Hematest?

Vitamin C interferes with the chemical reaction of the reagent and causes a false negative response, i.e., the test results come back negative even when blood is present in the stool sample.

thorazine

an antipsychotic agen used in treating manic-depression and hallucination

mellaril

an antipsychotic agent used in treating psychotic and severe depression

parnate

an MAOI usually usedin treating severe depression in patients who have failed to respond to other treatments

zoloft

considered the first line of treatment in panic-anxiety disorders and mild to moderate depresssion

a patient with hyperthyroidism is given phenobarbital to achieve what?

sedation

carafate

the action of this drrug is to line and protect the stomach, it is better able to do so if the medication can come in contact with the stomach

depo-provera

this medication suppresses endometrial bleeding

brevicon

this medication is a contraceptive agent

ANALGESICS

Drugs used to relieve or eliminate pain:
Aspirin
NSAID's e.g. Ibuprofen�
Morphine

Aspirin

Do not give together with other anticoagulants. Stop taking Aspirin
some days before surgery. Do not give to children with viral infection(Reye
syndrome)

NSAID's e.g. Ibuprofen

Take with food; contraindicated for people with GI
ulcers

Morphine

A respiratory depressant. It should be withheld if the respirations
are below 10

ANTI-CONVULSANTS:

Dilantin: Causes gum hyperplasia. Advice client to visit dentist frequently

ANTIINFLAMMATORY

Predisone: Causes Cushing like symptoms. Common side effects are
immunosupression(monitor client for infection), hyperglycemia

ANTI-COAGULANTS

Heparin:
Coumadin:

Coumadin:

Monitor pt's lab work�PT. Antidote is Vitamin K

Heparin:

Monitor pt's lab work-PTT. Antidote is protamine sulfate

ANTI-PARKINSONIAN

Cogentin:
Sinemet:

Cogentin:

Used to treat EPS

Sinemet:

Drug is effective when tremors are not observed

RESPIRATORY

Theophylline/Aminophylline: Side effects--Tachycardia

CARDIOVASCULAR

Digoxin (Lanoxin): Signs of toxicity: Pt will complain of visual change in colors.
They would also complain of loss of appetite.

ANTIHYPERTENSIVE (PRE-ECLAMPSIA)

Magnesium Sulfate: Monitor for deep tendon reflex and respiratory depression

DIURETICS

Hydrochlothiazide
Lasix
Aldactone

Hydrochlothiazide

Monitor potassium levels

Lasix

Monitor potassium levels

Aldactone

Potassium sparing

PSYCHOTROPICS

Lithium Carbonate:.
MAOI inhibitors:
Disulfiram (Antabuse)

Lithium Carbonate:.

Know therapeutic range (0.8 to 1.2mEq). Also know
symptoms of toxicity. Adequate fluid and salt intake is important

MAOI inhibitors:

Have dangerous food-drug interactions. Food with Tyramine
should be avoided. For example: aged cheese, wine etc.

Disulfiram (Antabuse)

Used for alcohol aversion therapy. Clients started on
Disulfiram must avoid any form of alcohol or they would develop a severe reaction.
Teach pt to avoid some over-the-counter cough preparations, mouthwash etc.

MATERNITY DRUGS

Oxytocin: Assess uterus frequently for tetanic contraction.

ANTIDOTES

Narcan
Calcium Gluconate
Vitamin K

Narcan:

Reverses the effects of narcotics

Calcium Gluconate:

Antidote for magnesium sulfate

Vitamin K

Antidote for Coumadin

Tegretol:

mood stabilizer - bipolar / anticonvulsant - carbamezepine

Atropine: What checks do you do before giving this drug ?

BP

Atropine

an antispasmodic that may be administered preoperattively to relax smooth muscles

Epogen:

Used in treating anemia because it increases RBC production.

Acyclovir:

anti-viral medication used in treating shingles.

arterial blood gases lab test measures what?

oxygen and pH levels

hemoglobin laboratory test measures?

rbc's and other things

glucose laboratory test measures?

glucose

electrolyte laboratory test measures?

measures potassium

gantrinsin

used in urinary tract infections

the most common angle for an intramuscular injection

90 degrees

patient on lasix may need

potassium supplements, to weigh self each wk. and change positions slowly to prevent dizziness, orthrostatic hypotension is a side effect of diuretic

terbutaline

beta1 direct agonist. Reduces premature uterine contractions

antihistamines adverse reactions

sedation, dry mouth, blurred vision, urinary retention

adrenergic stimulants adverse reactions

anxiety apprehension headache cerebral hemorrhage

adverse reactions to skeletal muscle relaxants

drowsiness, incoordination, GI upset

adverse reactions of uterine relaxants

heart palpitations, nausea, vomiting, headache

sign of digoxin toxicity

green yellow vision

digoxin is given to treat

elevated heart rate in chf client

lovenox is contraindicated in clients

on a kosher diet due to the pork content

lasix

helps decrease edema in the body

why do we give a client who is being treated for CHFa loading dose of digoxin

gives the patient an adequate blood level to achieve therapeutic relief as quick as possible

imodium

is used to treat diarrhea

apresoline

is an antihypertensive medication

vistaril

tused as an antiemetic or in higher doses as a tranquilizer

narcoan is given to overcome a narcotic overdose by

reverse CNS and respiratory depression

the desired effect of morphine is

pain relief

lithium

a medication used to treat bi-polar disorders

valium

a tranquilizer used to relieve anxiety and relax muscles

epinepherine

hormone that speeds up heart

inderal

the first beta blocker (trade name Inderal) used in treating hypertension and angina pectoris and essential tremor

angina pectoris

chest pain, which may radiate to the left arm and jaw, that occurs when there is an insufficient supply of blood to the heart muscle

histamine

amine formed from histidine that stimulates gastric secretions and dilates blood vessels, a regulating body substance released in excess during allergic reactions causing swelling and inflammation of tissues

aminophylline

used in bronchoconstriction broncial asthma and chronic obstructive pulmonary disease, and congestive heart failure, relaxes smooth muscle of the respiratory tract

beta-antagonist

decreases contraction of smooth muscle

decongestant

decrease the amount of mucus secreation from the bronchi ,reduces congestion and swelling of membranes, such as those of the nose and eustachian tube in an infection

adequate folic acid will helop to prevent

birth defects

whats the medication of choice for an alcoholic going through withdrawls

valium vit b1 and b12 and folic acid

tigan

A medication used to treat nausea and vomiting

admininster eye drops by dropping

lower conjuctiva

benadryl

is an anti-hystemine that decreases itching

lipitor is best given

at night this is when the body makes the most cholesterol

an important nursing interverntion while taking lasix

weights and vital signs. baseline is needed to determine the effectiveness of therapy

decadron

A drug in the steriod family that is used to decrease swelling in the brain.

heparin

prevents blood clotting

vancomycin

Anti-infective given for potentially life threatening infections(MRSA)

acyclovir

an oral antiviral drug (trade name Zovirax) used to treat genital herpes

clomid

a fertility drug that is used to stimulate ovulation and that has been associated with multiple births

an adverse reaction to atropine sulfate

tachycardia

oxytocin

stimulates uterine contraction

barbituates

drugs that depress the activity of the central nervous system, reducing anxiety but imparing memory and judgement induces sleepiness

side effects would be expected with elderly patients taking barbiturates

excitement confusion depression

absorption

the time it take the drug to be taken into the body to the time it enters the blood stream

distribution

the transport of drugs into the body

sublingual nitroglycerine

dilate blood vessels and increase circulation

streptase

lysis of thrombi in acute myocardial infarction

coumadin

an anticoagulant use to prevent and treat a thrombus or embolus

the drug of choice treating pancreatitis

demeral

cbc

a test ordered for suspected bleeding disorders

BUN

a test ordered for suspected renal disease

Blood glucose

a test ordered for suspected diabetes

cromolyn

drug used to prevent asthma attacks or decrease in allergic response

the appropriate size needle for intradermal injection

27 gauge 1/2 inch needle

the appropriate size needle for subcutaneanous injections

25 gauge, 5/8 inch needle

cipro

medication given for UTI, bone, joint, skin infectons

theophylline

a bronchodilator used to treat asthma and bronchitis and emphysema

the antidote for heparin

protamine sulfate

the antidote for coumadin

vit k

ace inhibitors

ACTION: prevent the conversion of angiotensin I to angiotensin II in the lungs USES: CHF, HTN , usually end in PRIL

side effects of ace inhibitors

A dry, hacking cough is COMMON. Hyperkalemia, renal tubular damage, decreased B/P, dizziness, nausea & diarrhea

NURSING IMPLICATIONSof ace inhibitors

: Administer 1 hr before meals to increase absorption

the antidote for anticoagulants

VITAMIN K

tolerance

the diminishing effect with regular use of the same dose of a drug, requiring the user to take larger and larger doses before experiencing the drug's effect

cumulative

the drug is not completly metabolized and is excreted before the next dose is given

synergistic

drugs that work together so the total effect is greater than if given seperatly

antitussive

any medicine used to suppress or relieve coughing

expectorant

drug that breaks up mucus and promotes coughing

urecholine

this drug produces smooth muscle contraction(bladder tone) and is used for abdominal and urinary retention

synthroid

A medication used to treat hypothyroidism

prilosec

antacid that suppresses acid secretion in the stomach best given on empty stomach

Kcl

Potassium chloride, it is often given to cardiac patients whose potassium is depleted by diuretic medications, such as Lasix.

tetracycline

comonly prescribed for acne vuglgaris
� Dental staining in children

silvadene

an antibiotic used topically in burn treatment

zocor

an oral lipid-lowering medicine administered to reduce blood cholesterol levels

fosamax

a medication used to treat osteoporosis, limit dairy products while taking this med

avandia

type 2 diabetes mellitus , oral diabetic drug doesn't produce more insulin, only gets glucose level to norm

specificgravity

measures a patients hydration status

the type of insulin used in an emergency situation is

regular

protease inhibitor is best taken

on an empty stomach 1hr b4 or 2hrs. after a meal

neupogen

stimulates the production of neutraphils

leukine

useful in treating patients with bone marrow transplant

gantrisin

is the most common anti-biotic to treat UTI's

what size needle would you use to administer vit. b12 to an average size person

22 gauge 1inch needle

cogentin

medication used to treat parkinson's disease

glucophage

an antidiabetic drug prescribed to treat type II diabetes the
action: decreases cellular resistance to insulin

characteristics of mild CNS depression

slow in initiating conversation

when should coumadin be administered

afternoon

troche

throat lozenger

adverse reactions to hyperthyroid medications

bronchospasms, iodism, weight gain and sleeping patterns increase

adverse reactions to hypothroid

headache and insominia

Aldactone is often prescribed for children with CHF because

Potassium sparing diuretic

corticosteriods is taken

with or after meals

decadron is best given

with a glass of milk to decrease gastric distress

what medication would be given fo an overdose of a cholinergic drug

atropine anticholenergic

patients who are receiving vancomycin by IV infusion should be assessed before administration and during for

hearing damage; this drug is both ototoxic and nephrotoxic

boniva

maintain adequate intake of calcium and vit. d

Librium

Anti anxiety

Xanax

Anti anxiety

Ativan

Anti anxiety

Vistaril

Anti anxiety

Equanil

Anti anxiety

Aluminum hydroxide

Anatacids

Milk of Magnesia

Anatacids

Maalox

Anatacids

Antacids Nursing Consideration

Interferes with absorption of antibiotics, iron preps, INH, oral contraceptives
Monitor bowel functions

Atropine sulfate

Antidysrhythmics

Lidocaine

Antidysrhythmics

Pronestyl (procainamide)

Antidysrhythmics

Quinidine

Antidysrhythmics

Isuprel (Isoproterenol)

Antidysrhythmics

-mycin AE

Ototoxicity, nephrotoxicity

cefaclor (Ceclor)

Cephalosporins

cefazolin (Ancef)

Cephalosporins

cephalexin (Keflex)

Cephalosporins

ceftriaxone (Rocephin)

Cephalosporins

cefoxitin (Mefoxin)

Cephalosporins

erythromycin

Macrolides (hepatotoxicity), confusion

clindamycin (Cleocin)

Macrolides (hepatotoxicity), confusion

Amoxicillin (Amoxil)

Penicillin (AE: stomatitis)

Amoxicillin/clavulanate (Augmentin)

Penicillin (AE: stomatitis)

Ampicillin

Penicillin (AE: stomatitis)

acetyl sulfisoxazole (Gantrisin)

Sulfonamides (AE: peripheral neuropathy, crystalluria)

co-trimoxazole (Bactrim)

Sulfonamides (AE: peripheral neuropathy, crystalluria)

sulfasalazine (Azulfidine)

Sulfonamides (AE: peripheral neuropathy, crystalluria)

doxycycline (Vibramycin)

Tetracyclines (AE: phototoxic reaction)

tetracycline (Panmycin)

Tetracyclines (AE: phototoxic reaction)

Furadantin (nitrofurantoin)

UTI (AE: asthma attacks, diarrhea);

Pyridium

UTI, urinary track analgesic
AE: HA, vertigo, change urine color to orange

Anti Impotence Contraindications

nitrates, alpha blockers

Anticholinergics Actions

dilates pupil, causes bronchodialtion and decreased secretions, decreases mobility and GI secretions (urinary retention)

propanthekine (Pro-Banthine)

anticholingergic

Atropine

anticholingergic

scopolamine (Scopace)

anticholingergic

Anticholinergics Uses

opthalmic exam, motion sickness, pre-operative

Anticholinergics contraindication

Contraindicated with glaucoma, paralytic ileus, BPH

Anticholinergics

o Anticholinergic:
� Mad as a hatter = cognitive dysfunction (drowsiness, confusion, agitation)
� Blind as a bat = blurred vision (mydriasis/pupils dilate), intraocular pressure, photophobia
� Red as a beet = vasodilation, tachycardia & BP followed by BP

Anticoagulant Heparin Nursing Considerations

Monitor clotting time or PTT
Normal 20-45 sec
Therapeutic level 1.5-2.5 times control
Antagonist- protmaine sulfate
Give SC or IV - Do NOT aspirate or Massage!

Anticoagulant Coumadin Nursing Considerations

Monitor PT
Normal 9-12 sec
Therapeutic level 1.5 times control
Antagonist- Vit K
vitC dec coumadin, vitE inc.
Monitor for bleeding
Give PO

food have vitK

green veg, pork, rice, yogurt, fish, milk, cheese

phenytoin (Dilantin)

Anticonvulsants (CNS depressant)aplastic anemia, gingival hypertrophy, ataxia; if give too quickly IV, can cause cardiac arrest. Have to give it alone

phenabarbital (Luminal)

Anticonvulsants (CNS depressant)aplastic anemia, gingival hypertrophy, ataxia

divalproex sodium (Depakote)

Anticonvulsants (CNS depressant)aplastic anemia, gingival hypertrophy, ataxia

carbamazepine (Tegretol)

Anticonvulsants (CNS depressant)aplastic anemia, gingival hypertrophy, ataxia

clonazepam (Klonopin)

Anticonvulsants (CNS depressant)aplastic anemia, gingival hypertrophy, ataxia

Anticonvulsants Side Effects

Respiratory depression, aplastic anemia, gingival hypertrophy, ataxia

Anticonvulsants Nursing Considerations

Don't d/c abruptly, Caution with use of meds that lower seizure thresholds (ex. MOAIs, antipyscotic)
No alcohol
Urine is pink/reddish

Marplan

MAOIs hypertensive crisis when taken with tyramine

Parnate

MAOIs hypertensive crisis when taken with tyramine (stiff neck, HA, palpatation, sweating)

Nardil

MAOIs hypertensive crisis when taken with tyramine

Antidepressants: MAOIs Side Effects

hypertensive crisis when taken with tyramine-containing foods (aged cheese, liver, beer, wine, bananas), photosensitivity

MAOIs Nursing Considerations

Do not take with cold medications or CNS stimulates

Paxil

SSRI (AE: anxiety, GI upset, change in appetite and bowel function, urinary retention)

amitriptyline (Elavil)

Tricyclics

imipramine (Tofranil)

Tricyclics

desipramine (Norpramin)

Tricyclics

Acetaminophen (APAP) / tylenol

Analgesic

Aspirin (ASA) / ecotrin, bayer

Analgesic

Azithromycin / zithoromax, zmax

Antiobiotic (Macrolides)

Amlodipine / norvasc

Calcium channel blocker

Alprazolam / xanax

Antianxiety (Benzodiazepine)

Atorvastatin / lipitor

Antihyperlipidemic (Statin)

Amoxicillin / amoxil

Antibiotic (Penicillin)

Amoxicillin + Clavulanate / augmentin

Antibiotic (Penicillin)

Atenolol / tenormin

Antihypertensive Beta Blocker (B1)

Albuterol / ventolin, ProAir, proventil

Respiratory (Bronchodilator)

Alendronate / fosamax

Osteoporitic

Amitriptyline / elavil

Antidepressant

Allopurinol / zyloprim

Antigout

Amphetamine + Dextroamphetamine / adderall

Neurologic / ADHD

Acyclovir / zovirax

Antiviral

Albuterol Ipratropium / combivent

Respiratory

Atometine / strattera

Neurologic / ADHD

Aripirazole / abilify

Anti-psychotic

Benazepril / lotensin

Antihypertensive

Bupropion / wellbutrin, zyban, budeprion

Antidepressant

Buspirone / buspar

Anxiolytic / Hypnotic

Benzonatate / tessalon perles

Cough Suppressant

Butalbital + APAP + Caffeine / fioricet

Analgesic

Bisoprolol / zebeta

Antihypertensive

Baclofenac / lioresal

Musculoskeletal

Clopidogrel / plavix

Cardiovascular

Citalopram / celexa

Antidepressant (SSRI)

Clonazepam / klonopin

Antianxiety (Benzodiazepine)

Cephalexin / keflex

Antibiotic (Cephalosporin)

Cyclobenzaprine / fexeril

Musculoskeletal (Muscle Relaxant)

Carvedilol / coreg

Cardiovascular (Beta blocker)

Codeine + APAP / Tylenol 3

Analgesic- CIII

Carisoprodol / vasotec

Antihypertensive

Clonidine / catapres

Cardiovascular

Celecoxib / celebrex

NSAID

Conjugated Estrogen / premarin

Hormone Replacement

Clindamycin / cleocin

Antimicrobial

Cefdinir / omnicef

Antibiotic (Cephalosporin)

Clotrimazole + Bethamethasone / lotrisone

Antifungal / Steroid

Clobestasol / Temovate

Corticosteroid

Clarithromycin / biaxin

Antibiotic (Macrolide)

Chlorpheniramine + / tussionex

Respiratory

Cefuroxime / ceftin

Antibiotic

Cetirizine / zyrtec

Antihistamine

Ciprofloxaxin / cipro

Antiobiotic (Quinolone)

Carisoprodol / soma

Muscle Relaxant

Cyclosporine / restasis

Immunosuppressant

Diphenhydramine / benadryl

Antihistamine

Diazepam / valium

Sedative / Antianxiety (Benzodiazepine)

Duloxetine / cymbalta

Antidepressant

Doxycycline / vibramycin

Antibiotic (Tetracycline)

Donepezil / aricept

Anti-Alzheimer's

Diclofenac / cataflam

NSAID

Diltiazem / cardizem

Antihypertensive

Divalproex / depakote

Anticonvulsant

Doxazosin / cardura

Antihypertensive

Dicyclomine / bentyl

Anti-spasmotic

Dutasteide / avodart

Urinary Tract Agent

Digoxin / lanoxin, digotek

Cardiovascular

Dexamethasone / decadron

Corticosteroid

Dimenhydrinate / dramamine

Antiemetics

Docusate Sodium / colace

Laxative (stool softener)

Diphenoxylate, Atropine / lomotil

Antidiarrheal

Esomeprazole / nexium

Antacid / Antiulcer (PPI)

Escitalopram / lexapro

Antidepressant (SSRI)

Ezitimibe, Simvastatin / vytorin

Antihyperlipidemic

Ezetimibe / zetia

Antihyperlipidemic

Eszopiclone / lunesta

Hyponotic

Enalapril / vasotec

Antihypertensive

Folic Acid / folvite

Mineral supplement

Furosemide / lasix

Antihypertensive Diuretic

Fluoxetine / prozac

Antidepressant (SSRI)

Fluticasone / flonase

Respiratory

Fluticasone, Salmeterol / advair

Respiratory

Fexofenadine / allegra

Antihistamine

Fluconazole / diflucan

Antifungal

Fenofibrate / tricor

Antihyperlipidemic

Fentanyl / duragesic

Analgesic (Topical)

Famotidine / pepcid

Antacid / Antiulcer

Finastreride / proscar

Urinary Tract Agent

Ferrous Sulfate / feosol

Antianemics

Gabapentin / neurontin

Anticonvulsant

Glimepiride / amaryl

Antidiabetic

Glyburide / diabeta, glynase, micronase

Antidiabetic

Glipizide / glucotrol

Antidiabetic

Gemfibrozil / lopid

Antihyperlipidemic

Guaifenesin / robitussin, mucinex, cheritussin

Respiratory (Expectorant)

Glyburide + Metformin / glucovance

Antidiabetic

Hydrocodone + Acetaminophen / lortab, vicodin, lorcet

Analgesic, Opiate

Hydrochlorothiazide (HCTZ) / Microzide

Diuretic (Thiazide)

Hydroxyzine Pamoate / vistaril

Antihistamine

Hydroxyzine HCL / atarax

Antihistamine

Hydroxychlorquine / plaquenil

Musculoskeletal

Hydralazine / apresoline

Antihypertensive

Hydromorphone / dilaudid

analgesic

Ibuprofen / motrin, advil

NSAID

Isosorbide Mononitrate / imdur, ISMO

Cardiovascular

Irbesartan / avapro

Antihypertensive

Ibandronate / boniva

Antiresorptive

Irbesartan/HCTZ / avalide

Antihypertensive

Ketoconazole / nizoral

Antifungal

Lisinopril / prinivil, zestril

Antihypertensive (ACE inhibitor)

Levothyroxine / synthroid, levoxyl, levothroid

Hormone Replacement

Lorazepam / ativan

Antianxiety (Benzodiazepine)

Lovastatin / mevacor

Antihyperlipidemic

Lansoprazole / prevacid

Antacid / Antiulcer (PPI)

Levofloxacin / levaquin

Antibiotic (Quinolone)

Losartan / cozaar

Antihypertensive

Lamotrigene / lamictal

Anticonvulsant

Latanoprost / Xalatan

Antiglaucoma (Opthalmic)

Losartan/HCTZ / Hyzaar

Antihypertensive

Lisdexamfetamine / vyvanse

Neurologic / ADHD

Lidocaine / lidoderm (patch), xylocaine (injection)

Anesthetic

Levetiracetam / keppra

Anticonvulsant

Metoprolol Succinate / Toprol XL

Antihypertensive Beta Blocker (B1)

Metoprolol Tartrate / lopressor

Antihypertensive Beta Blocker (B1)

Montlukast / singulair

Respiratory

Methylorednisolone / medrol (tablets), depo-medrol (injection), solu-medrol (injection)

Anti-inflammatory

Mometasone / nasonex

Corticosteroid

Methylphenidate / concerta, ritalin

Neurologic / ADHD

Metronidazole / flagyl

Antimicrobial

Metoclopramide / reglan

Gastrointestinal

Mirtazapine / remeron

antidepressant

Meclizine / antivert

Antivertigo

Mupirocin / bactroban

Antibacteria; (Topical)

Methotrexate / Rheumatrex

Anti-Rheumatic

Memantin / namenda

Anti-Alzheimer's

Methocarbamol / robaxin

Musculoskeletal

Moxifloxacin HCI / vigamox (opthalmic), avelox (tablets), avelox IV (injection)

Antibiotic (Quinolone)

Minocycline HCI / minocin

Antibiotic

Meperdine / demerol

Analgesic

Metformin / glucophage

Antidiabetic

Meloxicam / mobic

NSAID

Naproxen / aleve, naprosyn, anaprox

NSAID

Nitropfurantoin / furadantin

Antimicrobial

Nicin / niaspan

Antihyperlipidemic

Nifedipine / adalat, procardia

Antihypertensive

Nitroglycerin / NitroStat

Cardiovascular

Nabumetone Relafen

NSAID

Nortriptyline / pamelor, aventyl

Antidepressant

Omeprazole / prilosec

Antacid / Antiulcer (PPI)

Oxycodone + APAP / percocet, endocet, rocicet, tylox (capsule)

Analgesic, Opiate (Pain Relief)

Oxycodone / oxycontin

Analgesic, Opiate

Oseltamivir / tamiflu

Antiviral (Flu)

Olmesartan/HCTZ / benicar HCT

Antihypertensive

Omega-3/Fish Oil / lovaza

Antihyperlipidemic

Olanzapine / zyprexa

Antipsychotic

Oxybutynin / ditropan

Urinary Tract Agent

Prednisone / sterapred, orasone, deltasone

Corticosteroid

Pravastatin / pravachol

Antihyperlipidemic

Paroxetine / paxil

Antidepressant (SSRI)

Potassium Chloride / micro-k (capsule), k-dur (tablet), klor-con (tablet & liquid)

Potassium supplement

Pioglitazone / actos

Antidiabetic

Promethazine / phenergan

Anti-Nausea/ Anti-emetic

Pregabalin / lyrica

Anticonvulsant / Neurologic

Penicillin

Pen-VK

Phentermine / adipex, fastin

Appetitie Suppressant

Propanolol / inderall

Antihypertensive

Phenytoin / dilantin

Anticonvulsant

Phenazepyridine / pyridium, AZO standard (OTC)

Analgesic (Urinary)

Pantoprazole / protonix

Antacid / Antiulcer (PPI)

Prochlorperazine / compazine

Antiemetic

Prednisolone / prelone, pred forte (opthalmic)

Corticosteroid

Quetiapine / seroquel

Anti-psychotic

Quinapril / Accupril

Antihypertensive

Rosuvastatin / crestor

Antihyperlipidemic

Ranitidine / zantac

Antacid / Antiulcer

Ramipril / altace

Antihypertensive

Risperidone / risperdal

Anti-psychotic

Rabeprazole / aciphex

Antacid / Antiulcer (PPI)

Risedronate / actonel

Antiresorptive

Raloxifene / evista

Osteoporitic

Ropinirole / Requip

Parkinsons

Simvastatin / zocor

Antihyperlipidemic (Statin)

Sulfamethoxazole/trimethoprim / septra, bactrim

Antibiotic

Sildenafil / viagra

Erectile Dysfunction

Spironolactone / aldactone

diuretic

Sitagliptin / januvia

Antidiabetic

Sertraline / zoloft

Antidepressant (SSRI)

Sumatriptan / imitrex

Antimigraine

Tadalifil / cialis

Erectile Dysfunction

Tramadol / ultram

Analgesic

Trazodone / desyrel

Antidepressant

Tamsulin / flomax

Urinary Tract Agent

Temazepam / restoril

Sedative / Sleep Aid (Benzodiazepine)

Triamcinoline / nasacort, kenalog, aristocort

Corticosteroid

Tiotropium / spiriva

Anticholinergic

Topiramate / topamax

Anti-convulsant

Tolterodine / detrol

Urinary Anti-Spasmotic

Terazosin / hytrin

Antihypertensive

Tizanidine / zanaflex

Musculoskeletal

Tramadol + APAP / ultracet

Analgesic

Triamterene + HCTZ / dyazide (capsule), maxzide (tablet)

Antihypertensive

Venlafaxine / effexor

Antidepressant

Valsartan / diovan

Antihypertensive

Valacyclovir / valtrex

Antiviral

Verapamil / verelan, calan, isoptin

Antihypertensive

Varenicline / chantix

Smoking Cessation

Varedenfil / levitra

Erectile Dysfunction

Warfarin / coumadin

Anticoagulant (Blood Thinner)

Zolpidem / ambien

Hypnotic (sleep aid)

Beta Blockers

Decrease the activity of the heart. They block sympathetic stimulation of the heart and reduce systolic pressure, heart rate, cardiac contractility and output, so decrease myocardial oxygen demand and increase exercise tolerance.
-Used to treat angina, co

Coreg (Carvedilol)

Beta Blocker-olol
Used to treat heart failure, HTN, post heart attack.

Inderal (Propranolol)

Beta Blocker-olol
Used to treat tremors-angina, HTN, heart rhythm disorders, open heart circulatory conditions, prevent heart attack and reduce sensitivity and frequency of migraine H/A.

Atenolol (Tenormin)

Beta Blocker-olol
Used to treat angina, HTN, and treat/prevent heart attack

Metoprolol (Lopressor)

Beta Blocker-olol
Treat angina, HTN, prevent and treat heart attack

Alpha-Adrenergic Blockers

Relax certain muslces and help small blood vessels remain open. They work by keeping the hormone norepinephrine from tightening the muscles in the walls of smaller arteries and veins. Blocking that effect causes the vessels to remain open and relaxed. Thi

Minipress (Prazosin)

Alpha Adrenergic Blocker
Relaxes your veins and arteries so that blood can more easily pass through them.
Treat HBP and HTN.

Angiotensin-Converting Enzyme (ACE) Inhibitors

Help relax blood vessels.

Captopril (Capoten

Ace Inhibitor-pril
Used to treat HBP (HTN), CHF, kidney problems caused by diabetes and to imrpove survival after a heart attack.

Enalopril (Vasotec)

Ace Inhibitor-pril
Treat HBP (HTN) and CHF

Angiotensin 2 receptor blockers (ARBS)

Help relax blood vessels which lowers BP and makes it easier for your heart to pump blood.

Cozar (Iosartan)

ARBS
Keeps blood vessels from narrowing which lowers BP and improves blood flow. USed to treat HBP, also lower the risk of stroke, slow long term kidney damage in people with type 2 diabetes who also have HBP.

Diovan (HCTZ & Valsortan)

ARBS
Combo is used to treat HBP.

Calcium Channel Blockers

Disrupts the movement of calcium through calcium channels. Are used as anti-hypertensive drugs tot decrease BP.

Cardizem (Dilitazem)

CCB (-pine)
Relaxes muslces of heart and blood vessels. Treat HTN, angina, and certain heart rhythm disorders.

Norvasc (Amlodipine)

CCB (-pine)
Relaxes (widens) blood vessels and improves blood flow. Treat HBP, chest pain, and other conditions caused by coronary artery disease.
Used for adults and children > 6 y.o.

Anti-Platelet

Reduce blood clotting in an artery, vein, or heart.

Aspirin (Salicylate)

Anti-platelet
Reduces substances in the body that cuase pain, fever, and inflammation. Sometimes used to treat or prevent heart attacks, strokes, and chest pain.

Plavix (Clopidogrel)

Anti-platelet
Keeps the platelets in your blood from coagulating (clotting) to prevent unwanted blood clots that can occur with certain heart or blood vessel conditions.

Central Acting

Signal your brain and nervous system to relax blood vessels.

Aldoment (Methyldopa) & Catapres (Clonidine)

Central Acting
Lowers BP by decreasing levels of certain chemicals in blood. Allows blood vessels to relax and heart to beat more slowly and easily.

Anti-Anginal

Used in the treatment of angina pectoris,a s/s of ischemic heart disease.

Isosorbide

Nitrates
Dilates blood vessels. Treatment of angina.
Sublingual tablets

Anti-Arrythmic

Slow electrical impulses in the heart so that it can resume its normal rhythm and conduction patterns.

Amiodarone (Cordanone)

Used to help keep the heart beating normally in people with life threatening heart rhythm disorders of the ventircles. Also used to treat V tach and V fib.

Carbonic Anhydrase Inhibitor

Similar to diuretic.

Diamox (Acetazolamide)

Carbonic Anhydrase Inhibitor
Used to treat altitude sickness, CHF, and seizures.

Mineral/Electrolyte

Mineral/Electrolyte
Potassium Chloride
-Tx of hypokalemia
Beating of the heart
Don't stop abruptly
Dilute

Anti-arrhthymic

Anti-arrhythmic, Antedote-Digimmune Fab
Digoxin (Lanoxin)
Helps make the heart beat stornger and with a more regular rhythm. Used to treat heart failure, A fib, don't stop abruptly.
Side effects-uneven heart rate, tarry stools, blurred vision, confusion,

Anti-Coagulant

Coumadin-Antedote=Vitamin K
Reduces the formation of blood clots, Never take double dose. Performe regualr monitoring of INR (>4.0 great risk for bleeding, Target-2.0-3.0)
Heparin- Antedote=Protamine Sulfate
Only IV